ответственность врачей

десант

перемещено в Медицина



http://www.mk.ru/blogs/MK/2008/04/24/society/350250/
оправдали отца умершего ребенка, который пытался нанять киллера для убийства врача из-за которого ребенок умер.
только вот не понял, а врача то в итоге наказали или нет?
что вообще такое решение суда обозначает?

Kivar

Лучше бы ввели статью за невиновность, убившего врача... мента... прокурора... чиновника... не исполнивших своих обязанностей и содравших за это деньги.
Замочил, а тебе за это всенародный почет.

десант

жалко, что не нашел формулировки суда.
почему его невиновным признали?
ведь и доказательства все были и его признания и прочее.

Kivar

Закон - что дышло, куда повернул - туда и вышло... Ёбанная Россия...

Hartman

Простите, "плод стал задыхаться еще до родов" - я чего то не понял... плод ДЫШИТ находясь в матке ? Легкими ?
Медики, ау !

А вердикт мощный. Факт "заказа" установлен, признан судом (если верить стаье) - а чел-заказчик оправдан ?
Жесть. Родина слонов.

ЗЫ. Было у меня как то желание выпотрошить врача за небрежность. Но - заказывать не заказывал, все сам - "все, что доставляет мне удовольствия - я делаю сам"... До дела не дошло, правда - как то понял, отчего так и почему так получилось у врача... хорошо хоть так, а не хуже, уже молодец.
Просто помотрел, с кем он имеет дело каждый день и как он стоит у стола по 10 часов в сутки... стыдно мне стало.
Вместо потрошения принес ему бутыль коньяка и поблагодарил. 😊

finder00

Kivar
Ёбанная Россия...
годик отдохните от раздела, успокойтесь...

десант

Hartman
Простите, "плод стал задыхаться еще до родов" - я чего то не понял... плод ДЫШИТ находясь в матке ? Легкими ?
Медики, ау !

А вердикт мощный. Факт "заказа" установлен, признан судом (если верить стаье) - а чел-заказчик оправдан ?
Жесть. Родина слонов.

ЗЫ. Было у меня как то желание выпотрошить врача за небрежность. Но - заказывать не заказывал, все сам - "все, что доставляет мне удовольствия - я делаю сам"... До дела не дошло, правда - как то понял, отчего так и почему так получилось у врача... хорошо хоть так, а не хуже, уже молодец.
Просто помотрел, с кем он имеет дело каждый день и как он стоит у стола по 10 часов в сутки... стыдно мне стало.
Вместо потрошения принес ему бутыль коньяка и поблагодарил. 😊

интересна как раз обоснованность отказа в наказании.
а по врачам, слишком много сталкиваюсь с их ошибками и просто пренебрежением к здоровью пациента, чтоб в их положении входиь.
тем более в случаи родов, никакие ссылки на "с кем он имеет дело каждый день и как он стоит у стола по 10 часов в сутки" не пройдут.

АМАДЕУС

К сожелению плод может задыхаться
недавно горе у родственника было плод задохнулся в утробе матери
суть ребенок крутится и может передавить сам сеже пуповину (или как это называется) характерно на последних неделях беременности
вовсяком случае такой вердикт
Врачебный бардак это бардак страны когда за смешную зп работают не квалифицированные люди
ситуация патовая либо врача нет ,либо есть но хреновы, выбор из двух зол сами знаете, к хорошему не ведёт
Анищенко судить надо

десант

вот не надо про плохую зарплату у врачей роддомов в москве.
совсем она не смешная и люди квалиффицированные.
а вот отношение к своим обязанностям -отвратительное.

АМАДЕУС

десант
а вот отношение к своим обязанностям -отвратительное.
спорить не буду
в платных жопулижут факт, в обычных жопа, стравмой глаза (что считается надо лечить не медля) я москвич в своей клинеке был послан.
но зато поблату в подмосковной зделали всё не медля и бесплатно.

десант

к сожалению, в платных не заинтересованы в вашем выздоровлении. им выгодно вас долго лечить. и по дороже лечение прописать.

АМАДЕУС

десант
к сожалению, в платных не заинтересованы в вашем выздоровлении. им выгодно вас долго лечить. и по дороже лечение прописать.
тоже правда

shin-ap

Зав. отделением патологии беременности доктор мед. наук Липман элементарно кинул этих несчастных людей. Взял деньги и ничего не сделал. Видимо, не в первый раз. Наверное, надеялся, что дежурные врачи окажут помощь. А они, похоже, "встали в позу". Что тоже само по себе чудовищно. Какие бы не были отношения между врачами, в т.ч. "товарно-денежные", нельзя сводить счёты, используя жизнь больных.
Лично я жалею, что Липмана не грохнули. Видел недавно по ТВ его самодовольную морду.

десант

да даже без патологии.
роддом N27 в москве. еще перед родами оглашают родственикам негласные тарифы по оплате медуслуг.
то есть в карман вынь и положь деньгу. в итоге доходило до 25 000 рублей за роды, которые ты должен неофициально оплатить. причем без всяких гарантий и обязательств со стороны медиков.

Hartman

десант

интересна как раз обоснованность отказа в наказании.
а по врачам, слишком много сталкиваюсь с их ошибками и просто пренебрежением к здоровью пациента, чтоб в их положении входиь.
тем более в случаи родов, никакие ссылки на "с кем он имеет дело каждый день и как он стоит у стола по 10 часов в сутки" не пройдут.

Ну, это вопрос персоны и ее отношения к мысли "Неужели весь мир не крутиться вокруг меня ?"...
Обоснованность - вердикт суда присяжных, я так понял.

Васёк

Я всегда считаю наших врачей ветеринарами. Сохраняет много времени, денег и нервов.
Когда жена рожала, подключил к делу всех родичей - дохтуров, проплатил всё, что можно, на родах присутствовал, всё лично контролировал. Всё хорошо прошло в итоге.
По детским заболеваниям мы с женой стали лучше соображать, чем в поликлинике.
Сами знаем, какие таблетки от головы, какие от попы.
В сложных случаях консультируемся у друзей - квалифицированных врачей. Друзья-то уж не посоветуют неправильную таблетку, только чтобы заработать на нас.
В общем, "спасение утопающего" в полной мере.

------------------
-Если ты подумаешь, что эта хреновина - не транклюкатор,
это будет последняя мысль в твоей чатланской башке (с) дядя Вова

Hartman

десант
да даже без патологии.
роддом N27 в москве. еще перед родами оглашают родственикам негласные тарифы по оплате медуслуг.
то есть в карман вынь и положь деньгу. в итоге доходило до 25 000 рублей за роды, которые ты должен неофициально оплатить. причем без всяких гарантий и обязательств со стороны медиков.

Ничто не обходится так дорого, как бесплатная медицина... 😊
В моем случае бесплатная медицина (там выбора не было, по скорой привезли) - обошлась ровно в два раза дороже, чем то же членовредительство в платной клинике.

Почемушка

Hartman
Простите, "плод стал задыхаться еще до родов" - я чего то не понял... плод ДЫШИТ находясь в матке ? Легкими ?
Медики, ау !
неправильная формулировка. Плод может погибнуть от недостатка кислорода передаваемого от матери посредством пуповины. Связано это или с измнение положение плода и передавливанием пуповины, или же мать плохо дышит по какой то причине, например сама уже потеряла сознание или отслойка плаценты раньше времени и прочие проблемы..

Hartman

Почемушка
неправильная формулировка. Плод может погибнуть от недостатка кислорода передаваемого от матери посредством пуповины. Связано это или с измнение положение плода и передавливанием пуповины, или же мать плохо дышит по какой то причине, например сама уже потеряла сознание или отслойка плаценты раньше времени и прочие проблемы..

Ясно, я так и думал... Спасибо.

десант

Hartman

Ну, это вопрос персоны и ее отношения к мысли "Неужели весь мир не крутиться вокруг меня ?"...
Обоснованность - вердикт суда присяжных, я так понял.

вот я про это и спрашиваю
как можно вынести оправдательный вердикт при неопровергнутой докозательной базе?
какое основание суд записал в вердикте?


десант

Васёк
Я всегда считаю наших врачей ветеринарами. Сохраняет много времени, денег и нервов.
Когда жена рожала, подключил к делу всех родичей - дохтуров, проплатил всё, что можно, на родах присутствовал, всё лично контролировал. Всё хорошо прошло в итоге.
По детским заболеваниям мы с женой стали лучше соображать, чем в поликлинике.
Сами знаем, какие таблетки от головы, какие от попы.
В сложных случаях консультируемся у друзей - квалифицированных врачей. Друзья-то уж не посоветуют неправильную таблетку, только чтобы заработать на нас.
В общем, "спасение утопающего" в полной мере.

анологично, постоянно жалею об отсутствии полноценного медицинского образования.
проблема еще в том, что государство нас насильно медикам в руки отдает. особенно то что детей касается.

Hartman

десант
вот я про это и спрашиваю
как можно вынести оправдательный вердикт при неопровергнутой докозательной базе?
какое основание суд записал в вердикте?

"Мы голосовали сердцем", видимо... или еще каким голосовательным органом...

десант

и что такое прокатит?
неужели надеются, что прокуратура не обжалует решение?
или там указка сверху пришла?

Hartman

Писал и повторюсь - проблема о двух концах.
Первое - вскрытие умершего больного проводит патолог больницы, числящийся в том же шатате, что и, допусти, хирург или геник или еще кто то.
И получающий там же зряплату.
Нет такого зверя, как "коронер" - независимый эксперт, в обязанность которого входит и экспертиза причин смерти больных.
Поэтому больничный паталог напишет то, что надо, а не то что есть - в спорных случаях. А заодно может и уничтожить улики.
Это раз.

Два. Жесткая ответственность врачей влечет за собой перестраховку врачей при выборе лечения. Как в Германии - есть компьютер, есть меню с симптомами. И каждый свой пук врач подтверждает рекомендациями системы. Не дай Боже что - у него есть стопудовая отмазка. Документированная. Одобренная Менздравом. И т.д. и т.п.
А это сильно влияет на скорость (положено в таком порядке делать анализы - вот и делаем... а что помер в процессе - дык есть инструкция), качество и бюрократизирует систему еще больше. В результате вырабатывается бюрократия, на которую и переноситься ответственность.

ЗЫ. Рассказывал, как знакомого не брали в больничку - гематома мозга запущенная (травки пил и мегнит привязывал к башке - чудак) - мол, иннах, помрешь тут, а нам потом жопы надерут... все равно помрешь, но лучше - чтобы не на столе и вне здания боьлнички...

Daina

Hartman

"Мы голосовали сердцем", видимо... или еще каким голосовательным органом...

+1
Я фигею от суда присяжных в отечественном исполнении...

Читала про это в МК и думала: праздник будет на форуме - у здешних сторонников самосуда.

Hartman

десант
и что такое прокатит?
неужели надеются, что прокуратура не обжалует решение?
или там указка сверху пришла?

Ну, сколько у нас раз Ульмана судили судом присяжных ? 😊

десант

дела разные. у Ульмана хотя бы каие то оправдательные причины были и следствие не опровергло их (там же судили офицера, передавшего приказ).
а здесь то вообще ничего этого нет.
получается за мотив оправдали?

Daina

десант
получается за мотив оправдали?

Именно так.

Mahaon007

По отечественным дохтурам. Первое, что пришло вспомнилось.
Начало 90х. Пацана сбивает машина. Через 300 метров - больница.
Пока ждали скорую, сбегали туда. Не вышла не одна падла. Скоряга приехала через 2! часа. Пацан умер через час после вызова.
Через несколько лет. Двое поломанных. У одного тяжелый сотряс, у второго открытый перелом. Оба в кровище, один периодически плывет. Оба заходят рядом в больницу, им предложено обратится в травмопункт. Ни остановить кровь, ни вколоть обезболивающие до приезда - них.. я. Так и сидели на тротуаре, машину ждали.
Случай с роженицей. Дотерпели красавцы до последнего, при схватках вызвали скорягу.
Приехаший лепила осмотрел её, и выразил предположение, что болевые симптомы это результат нарушений правил личной гигиены женщины. Проще говоря надо чаще подмываться. За что был хорошенько отпзж.. н, и спущен с лестницы. Обещанные им гости в погонах так и не появились. Жаль.
В двух последних случаях помощь просилась в обмен на деньги. Как полагается. Пох. Никому, ничего не надо. То есть взять бабло в случае не требующем экстренности, или за какую то услугу - пожалуйста. Когда реально надо действовать руками и брать ответственность на себя - х..
Это на гражданке. К военным докторам у меня отношение другое. Хотя и там дерьма хватает.
Здесь же.. Не знаю ни одного положительного результата связанного с ними.
Только пиз.. еть, тянуть бабло, и жаловаться на хреновую жизнь.
Хотя про доход обычных сотрудников на выездах я знаю из первых уст.
Желающим могу назвать сколько в сутки минимум. Уверяю, немало. Гаишники правда побольше имеют.
Случаев схожих с приведенными у меня много. Всё не напишешь.
Про коммерческую медицину - развод ещё веселее. Могу рассказывать долго.
Но между делом замечу. Знакомый , наемный работник одной частной клиники. Недешовой, сразу скажу. И рекламу её многие видели. В должности коммерческого директора. Свою жену отправил рожать в обычный роддом. Ну под присмотром и за бабло ессно. На свою клинику положил толстый и продолговатый предмет. Ибо квалификация и реальные возможности персонала - анекдотичны были.

десант

у военных не лучше.

вообще на тему мдецины можно долго говорить, но я тему немножко для жругого открыл.
здесь интересный судебный прецендент получается.

Mahaon007

Прецендент. Но вероятно учитывая личность обвиняемого, отсутствие результата его действий, очевидное раскаяние и отказ от планирования дальнейших злодейств сыграли на решение суда. Может и ещё что-то сыграло. 😊
Хотя согласен. Не типично.
А может судья схож во мнение с моим. Кто знает.
Насчет темы медицины.

десант

это все могло повлиять только на тяжесть наказание, дали бы условку и все.
а здесь признали -невиновным.
то есть получается, что суд посчитал, что он ничего и не готовил.

Daina

Mahaon007
А может судья схож во мнение с моим. Кто знает.
Насчет темы медицины.

Видимо так.

Я читала про этот случай, когда мужика только что задержали. В т.ч. интервью с ним.
Он не раскаивался. На публику, журналисту - говорил, что жалеет только о том, что не получилось. Очень даже идейный мститель.

Сердцем голосовали присяжные, точно. Прецедент.

десант

все равно посадят.
прокуратура такого не отпустит.

Hartman

Daina
...

Сердцем голосовали присяжные, точно. Прецедент.

Вот весело будет - выпустят этого штриха на волю - а он к генику придет на работу и шпокнет его. Купит себе шпокун - - "шпок-шпок-шпок"...
А потом в тюрьму... или опять присяжные оправдают ?

Daina

десант
все равно посадят.
прокуратура такого не отпустит.

Как и в случае с гибелью человека от бродячих собак, есть повод для осторожного оптимизма: может, после ЭТОГО власть предержащие задумаются об очевидности идиотизма?

Daina

Hartman

Вот весело будет - выпустят этого штриха на волю - а он к генику придет на работу и шпокнет его. Купит себе шпокун - - "шпок-шпок-шпок"...
А потом в тюрьму... или опять присяжные оправдают ?

Первое что пришло в голову: продолжит, вдохновившись оправданием.

Надо на досуге в УК порыться - можно ли в таком случае что-то предъявить присяжным.

десант

а зачем ему вдохновлятся.
у человека со смертью ребенка смысл жизни пропал.
ребенок умер, других они уже не родят. что он теряет то.
если решился мстить, то он так и будет.
посадят, выйдет и отомстит.

Daina

десант
а зачем ему вдохновлятся.
у человека со смертью ребенка смысл жизни пропал.
ребенок умер, других они уже не родят. что он теряет то.
если решился мстить, то он так и будет.
посадят, выйдет и отомстит.

Смысл жизни - тонкая материя. Можно долго на эту тему говорить.
А насчет вдохновения...
Отцовство не состоялось, замкнуло - смысл жизни с врачом разобраться. Нафига он живет, если ребенок умер - пусть не живет тоже.
В камере посидел - был повод призадуматься. Насчет смысла жизни (хотя бы о жене - она-то живая).
И вдруг - оправдание. Не виноват! Доказано, и сам вину признал - а не виноват! Значит - прав. Значит - можно. Присяжные так решили. Глас народа.
Как-то так.

Mahaon007

Daina
Он не раскаивался. На публику, журналисту - говорил, что жалеет только о том, что не получилось. Очень даже идейный мститель.

Сердцем голосовали присяжные, точно. Прецедент.

Честно ответил. Не юлил. Психика наверняка на реактиве была тогда.
А что не получилось, прямо указывает - не в теме человек. Без опыта. 😀
"МанИак-душегуб-мерзавец" был бы - получилось.
А так.. Доведенный расп... ями обыватель.
Оттого може и сердцем порешали. Со скупой, как говориться на щеке.. 😊
Вот весело будет - выпустят этого штриха на волю - а он к генику придет на работу и шпокнет его. Купит себе шпокун - - "шпок-шпок-шпок"...
А на кой чорт пардон, ему геник? Или дядько решивший отомстить конкретному лицу на весь мир зуб нарисовал? Что геник тогда ему. Самолет - небо - Белый дом на снижении. Что мелочиться то? 😀
а зачем ему вдохновлятся.
у человека со смертью ребенка смысл жизни пропал.
ребенок умер, других они уже не родят. что он теряет то.
если решился мстить, то он так и будет.
посадят, выйдет и отомстит.
Здесь все не так просто..
Может и так. А может и нет.
Никто из присутствующих ему в душу не залезет.
И гадать, как он ситуацию разрулит на момент в будущем - не стоит.
Может и опез.. ла того успокоит, может себя. А может и родит-усыновит другого. Его право, и его дело. Один вон в Швейцарию за таким делом ездил. И ничего. Уважаемым человеком стал у себя потом.

Hartman

Mahaon007
А на кой чорт пардон, ему геник? Или дядько решивший отомстить конкретному лицу на весь мир зуб нарисовал? Что геник тогда ему. Самолет - небо - Белый дом на снижении. Что мелочиться то?

А я знаю ? 😊 Невзлюбил он геника то, убью я тебя, говорит, геник. Жди. Только найму людей, объявление дам в газетку, а то у самого кишка тонка и от крови мутит...

Mahaon007

О как. 😊 Прям таки и невзлюбил геника? И даже сказал ему правду-матку такую? Нехорошо то как.. С геником.. Зря. 😊 Нельзя так.
Всё, начинается стеб.
Но почему кажется, что айболит тот, ну совсем не по гинекологии болячку себе нажил.
Это к проктологам ближе. Баальшоой такой гемморой на подходе у него.
Не ту гинеколог себе профессию выбрал. 😊 Не ту.

shin-ap

Когда я был студентом (70-е годы 20 века), то был такой случай: на территории больницы нашли убитого врача акушера-гинеколога. В задницу ему был вставлен деревянный кол. Конечно, никого не нашли. Но было мнение, что он брал деньги и плохо делал криминальные аборты. Другая версия - "голубая".
Хочу обратить внимание на то, что пострадавший и доктор - лица одной древне-исторической национальности. А методы разборок - НАШИсткие 😊.

Mahaon007

Таки да. Один должен был заговорить другого до состояния невменоза. 😀
Или, шо тоже катит, пригласить третьего, специально обученного знаниям, и уже с его помощью убедить оппонента таки подписать неоднозначное признание его, поца некомпетентности. Что являлось бы косвенным доказательством в дальнейших спорах за "последнюю истину".
А так.. Поцы. Оба. 😀 😀 😀

Dr. San

Если уж речь зашла о врачах...

1. Государство требует (и нацпроект "здоровье" это предусматривает) чтобы медицинская помощь была:
- качественная
- бесплатная
- досутпная.

Помните анекдот про... "Мы можем выполнить ваш заказ быстро, дешево, качественно. Выбирайте любые два пункта."

Если поувольнять всех, кто не может работать качественно - медицинская помощь станет малодоступна населению. И сейчас укомплектованность официальной медицины кадрами не превышает 60%. Нормально работать могут 15-20% врачей. Далее считайте.

2. От размера зарплаты качество медицинской помощи не зависит. Сколь ни повышай зарплату бракоделу и халявщику - лучше он работать не будет.

3. В моей больнице в 90-х был развернут эксперимент - "НХМ в здравоохранении". Тогда зарплата врачей была поднята до ДОСТОЙНОГО уровня, но поставлена в зависимость от качества работы. Плохие врачи начали увольняться, хорошие - работать лучше и больше. Но государству это было не нужно. Эксперимент зарубили на корню, ввели медстрахование и уравниловку. Хорошие врачи поуходили в частную медицину и столичные клиники, где платили больше, плохие остались. Теперь, если опять попытаться ввести систему зависимости приличной зарплаты от качества работы понадобится лет 20-30...

4. Врачебные ошибки всегда были, есть и будут. Без системы материального стимулирования ХОРОШЕЙ работы, одними "кнутами", хорошо работать заставить невозможно. Это позволило бы уменьшить количество ошибок, но не извести их совсем. Это позволило бы обеспечить приток в официальную медицину нормальных специалистов. Просто чтобы придти в муниципальную больницу из частной клиники, врач должен знать, что будет получать столько же, если будет работать качественно. Средняя зарплата врача 15-25 тысяч. В частной клинике - 100-150... Считайте сами...
Но даже если государство такое решение примет, на возврат в официальное здравоохранение нормальных врачей понадобится лет 20-30...

5. Относительно обсуждаемого случая. Отцу ребенка 38 лет. Матери вряд ли меньше 33-35... Заведомо неблагоприятная комбинация, череватая осложнениями. В ситуации полного здоровья и абсолютной чистоты экологии - фигня. Но в данном случае - вряд ли...
Беременность и роды рискованы изначально. Так что катить бочку на врачей без конкретных фактов прреступной халатности... Некомильфо...

MNK

десант
все равно посадят.
прокуратура такого не отпустит.

Конечно не отпустит! Подадут протест и на пересмотр.. Вот пожалуйте, дело "оправданных" ментов вернули: http://guns.allzip.org/topic/103/315992.html

MNK

Daina

Первое что пришло в голову: продолжит, вдохновившись оправданием.

Надо на досуге в УК порыться - можно ли в таком случае что-то предъявить присяжным.

Насчёт присяжных не скажу, а вот судьям вполне можно предъявить:

Статья 305. Вынесение заведомо неправосудных приговора, решения или иного
судебного акта.

1. Вынесение судьей (судьями) заведомо неправосудных приговора, решения
или иного судебного акта -
наказывается штрафом в размере от пятисот до семисот минимальных размеров
оплаты труда или в размере заработной платы или иного дохода осужденного за
период от пяти до семи месяцев либо лишением свободы на срок до четырех лет.
2. То же деяние, связанное с вынесением незаконного приговора суда к
лишению свободы или повлекшее иные тяжкие последствия, -
наказывается лишением свободы на срок от трех до десяти лет.

Там-же в комментариях:

1. Под неправосудным приговором (решением, определением, постановлением)
понимается любой судебный акт, который вынесен не в соответствии с обстоятельствами
дела и тем самым противоречит требованиям материального или (и) процессуального
закона. По уголовному делу это может быть несоответствие приговора полученным
доказательствам, неправильная квалификация преступления, несправедливая мера
наказания и др. По гражданскому делу - неверные отказ или, напротив, удовлетворение
искового требования, неправильное определение суммы, подлежащей взысканию,
и др. Надо полагать, что статья распространяется и на решения арбитражных
судов.
Впрочем, ведь присяжные, они потому присяжные, что присягу дают честно и непредвзято рассмотреть дело. А не "голосовать серцем" 😛

Daina

MNK
Насчёт присяжных не скажу, а вот судьям вполне можно предъявить:

Ясно. Читать в выходные буду не УК, а УПК. Про присяжных - чем они от судей отличаются.

KirillD

Сами же врачей и научили брать взятки. Иной раз никто не просит, а дотошная мамочка сует денежку. Избаловали, вот и страдают те кто хочет жить честно или не имеет денег на взятки.

А должно быть так, попытка вымогательства, заявление, скорый суд, тюрьма, огласка в СМИ. И собственность такого врача конфискануть. Все до копейки. Квартиру, дачу, машину. и также карать тех кто дает взятки, еще боле жестоко, за подпитку коррупционной системы.

десант

вперед на баррикады.
мы новый и светлый мир построим.

KirillD

не баррикады! А порядочность и честность людей, и четкая работа правоохранительных органов. И ВСЕ. Два необходимых звена.

десант

и как вы этого добьетесь?
где будем честных людей искать?

Daina

десант
и как вы этого добьетесь?
где будем честных людей искать?

Ню... Два уже есть: Калоев и этот, из сабжа - Брежо.
Министр и зам.

Hartman

Daina

Ню... Два уже есть: Калоев и этот, из сабжа - Брежо.
Министр и зам.

О ! Калоева - в президенты ! 😀

михон

KirillD
А порядочность и честность людей, и четкая работа правоохранительных органов. И ВСЕ. Два необходимых звена.
😀 А Вы, батенька, утопист

Шилохвость 1

Порядочность и честность людей в любых областях деятельности это почетно.
Для Хартмана сообщаю, что дело Ульмана живет, сам он гдето ходит... по просторам... и еще с 2-мя товарищами.

Дог

порядочность и честность людей, и четкая работа правоохранительных органов. И ВСЕ. Два необходимых звена.
А может проще коммунизм?

------------------
Lupus lupo homo est

vasia2009

>Про присяжных - чем они от судей отличаются.

они есть народ ака единстный источник закона и власти - че решили то и есть закон...

а че юзеры сами то не рожали - разучились типа... и гибель потомства есть естессная плата за прожывание в развитой стране - она ж спициально делает шоб пара юзеров не могла произвести неск десятков потомков за время жызни... а если захочет - то не сможет... а если сможет - потомство должно сдохнуть естессным и необидным образом... ну из-за ограничения на 1..2 потомка - иногда и одново не получаеццо сделать - ну так уж есть...

Doctor_D

5. Относительно обсуждаемого случая. Отцу ребенка 38 лет. Матери вряд ли меньше 33-35... Заведомо неблагоприятная комбинация, череватая осложнениями. В ситуации полного здоровья и абсолютной чистоты экологии - фигня. Но в данном случае - вряд ли...
Беременность и роды рискованы изначально. Так что катить бочку на врачей без конкретных фактов прреступной халатности... Некомильфо...
+1
Кстати, никто не мещал этому типу решить все по закону (за те же деньги, что и на "заказ" собрался потратить).

А так - зашибись ситуация: если за каждого умершего родственники пациента будут устраивать самосуд (в медицине у нас каждый слесарь охренеть как компетентен!), ничем хорошим это не кончится.

Интересно, что о "убийцах в белых халатах" пресса писать очень любит, а вот о попытках "разводов" со стороны пациентов - почему-то нет. Несправедливо.

KirillD

Если тот врач уж такой какой он есть, то есть взятку взял и не сделал нормального результата, то по понятиям обязан отдать деньги. Судя по всему не отдал.

Mahaon007

Doctor_D
[QUOTE][B]Кстати, никто не мещал этому типу решить все по закону (за те же деньги, что и на "заказ" собрался потратить).
Акуенно. Решить по закону за деньги? Сами то поняли, что сказали?
Или вы про дороговизну "компетентности" адвокатов намекали? Так компетентность та в РФ не знаниями законов определяется. Особо по таким мелким вообщем то делам. 😀 Причем такого "лоха", с точки зрения таких адвокатов просто грех не кинуть. Это я вам ответственно заявляю.
в медицине у нас каждый слесарь охренеть как компетентен!), ничем хорошим это не кончится.
Вот, вот. В медицине недослесарей - до и больше. И каждый "компетентностью" Парацельса заткнет. Гиппократы районного масштаба блин.
Если и есть нормальные специалисты - их крайне мало. И не по ним можно судить по общей массе. Тут уж без обид.

KirillD
Если тот врач уж такой какой он есть, то есть взятку взял и не сделал нормального результата, то по понятиям обязан отдать деньги. Судя по всему не отдал.
Я вам байку расскажу. Короткую. Один художник нарисовал картину.
Зеленый конь с восемью ногами, и одним крылом.
На выставке-продаже его спросили:
- Что это?
Он ответил:
- Я ТАК вижу.
-Если ТАК видишь, на кой хер рисуешь?

А жизнь человека не картина. И если херово видишь - иди другим займись. А допустил подобный косяк, с раскладом таким, готов будь ответить. И не баблом уже. А как с тебя спросить решат.
Это про понятия вами упомянутые.
А так, мутное дело конечно.
Но на развод со стороны мужичка не похоже.
По ходу неправы с ним были. Вот он и затаил недоброе.

Добрыня Никитич

Hartman
До дела не дошло, правда - как то понял, отчего так и почему так получилось у врача... хорошо хоть так, а не хуже, уже молодец.
Просто помотрел, с кем он имеет дело каждый день и как он стоит у стола по 10 часов в сутки... стыдно мне стало.
Вместо потрошения принес ему бутыль коньяка и поблагодарил. 😊

+100!
Я лично всегда уважал и уважаю в людях умения: взглянуть на проблему объективно и понимать мотивы других людей. Иначе часто можно наломать дров на ровном месте...


Добрыня Никитич

десант
анологично, постоянно жалею об отсутствии полноценного медицинского образования.

Я иногда тоже об этом задумываюсь. Но гораздо больше я жалею об отсутствии полноценного юридического образования. Потому как у нас в стране, при существующей пропасти между законодательством и правоприменительной практикой, необходимо хорошее знание своих прав и возможностей. Это первый вариант преодолеть эту пропасть. Есть, конечно, второй - ещё более надежный. Мостик из бумажных купюр. 😊

Добрыня Никитич

Mahaon007
Вот, вот. В медицине недослесарей - до и больше. И каждый "компетентностью" Парацельса заткнет.

Махаон, как всегда, в точку. 😊
Хорошо, когда здоровье позволяет очень редко посещать вачей... Хотя, по-любому - через несколько десятков лет большинству из нас придется начать с этим сталкиваться. А в случаях родов у жены, хронических заболеваний, стоматологии - намного раньше...
Я лично сам хрен знает сколько раз - сам, через родственников или друзей - сталкивался с полнейшей некомпетентностью медиков. 😞

Dr. San

KirillD
Если тот врач уж такой какой он есть, то есть взятку взял и не сделал нормального результата, то по понятиям обязан отдать деньги. Судя по всему не отдал.

Ну как Вы до сих пор не поймете - если уж зарплату врачам (официально !!!) платят не за результат, а за "предоставление услуги"...

Взятки думаете берут за гарантированный результат?

Doctor_D

Акуенно. Решить по закону за деньги? Сами то поняли, что сказали?
Или вы про дороговизну "компетентности" адвокатов намекали?
Разумеется. Услуги адвоката, организации экспертиз и прочее - стоят денег. Не думаю, что сильно дороже услуг киллера.
А результат был бы куда лучше во всех смыслах.
Вот, вот. В медицине недослесарей - до и больше. И каждый "компетентностью" Парацельса заткнет. Гиппократы районного масштаба блин.
Если и есть нормальные специалисты - их крайне мало. И не по ним можно судить по общей массе. Тут уж без обид.
Ну, в любом случае, плохой врач разбирается в медицине лучше, чем хороший слесарь, не так ли? 😊
Проблема плохих докторов решается просто: обращайтесь к хорошим. 😛


Ну как Вы до сих пор не поймете - если уж зарплату врачам (официально !!!) платят не за результат, а за "предоставление услуги"...

Взятки думаете берут за гарантированный результат?

+1.
Кстати, хороший врач никогда не будет давать "гарантию результата". Если он это делает - явно купил диплом и халат. 😛

Mahaon007

Разумеется. Услуги адвоката, организации экспертиз и прочее - стоят денег.
Про связь результативности и компетентности уже писал. Повторюсь, крупная сумма потраченная на адвоката, вовсе не свидетельствует о его профессиональных навыках. Есть гораздо более действенные вещи для того или иного решения в суде. Они то и будут задействованы дорогим адвокатом в дальнейшем. Вы о таком правосудии? А назначение дорогих экспертиз, восстановление хода событий, сбор доказательной базы - оставьте честным, хорошим но более "бюджетными" адвокатам. Хотя подобными спектаклями могут славится и дорогие. Особенно, когда к их услугам прибегает несерьезный, "одноразовый" клиент. Набравший из последних сил нужную сумму. Определение таких клиентов тоже приводил. 😊
А результат был бы куда лучше во всех смыслах.
Откуда такая уверенность? Есть статистика неуспешности "альтернативных" методов? 😊
А статистика проваленных частными обвинителями аналогичных дел - гора.
Ну, в любом случае, плохой врач разбирается в медицине лучше, чем хороший слесарь, не так ли?
Не вопрос. Но вреда от плохого слесаря несравнимо меньше, чем от плохого врача. Исправить работу самого плохого слесаря без особого труда сможет слесарь хороший. Чего не скажешь о врачах. Оттого и ответственность у них разная должна быть. Не должно быть плохих врачей! Это утопия, понятно. Но когда плохих становится больше хороших - вовсе кошмар.
Проблема плохих докторов решается просто: обращайтесь к хорошим.
Проблема пищевых отравлений некачественными продуктами питания, решается покупкой заранее качественных продуктов. Ну-ну. 😊
Кстати, хороший врач никогда не будет давать "гарантию результата".
В серьезных делах - да. А в мелочи, вроде удаления вросшего ногтя - почему бы и нет? И то, при отсутствии дальнейших побочных эффектов.
Или когда пациент сам не соблюдал предписания. Тут понятно, и никаких претензий быть не может.
Если он это делает - явно купил диплом и халат.
Ни чуть не лучше фразы вроде: "Медицина - полунаука, полуискусство".
Причем фразу эту любят произносить, как и с купленными, так и с нормальными дипломоми. 😊

Doctor_D

Про связь результативности и компетентности уже писал. Повторюсь, крупная сумма потраченная на адвоката, вовсе не свидетельствует о его профессиональных навыках.
Адвокаты, как и врачи бывают хорошие и плохие. Сумма, необходимая для оплаты их услуг с квалификацией напрямую не связана.
Есть гораздо более действенные вещи для того или иного решения в суде. Они то и будут задействованы дорогим адвокатом в дальнейшем. Вы о таком правосудии?
А оно у нас такое, какое есть. Впрочем, и не слишком дорого можно решить вопрос (просто дольше и нервов больше потратишь) если есть за что цепляться. (Случай из личной практики: лет 10 назад моя сестра с помощью весьма бюджетного адвоката и пишущей машинки развалила заказное дело против мужа до такой степени, что на суде прокурор отказался поддерживать обвинение (изначально вменялось аж 8 статей, 7 из них - особо тяжкие).)
Особенно, когда к их услугам прибегает несерьезный, "одноразовый" клиент. Набравший из последних сил нужную сумму. Определение таких клиентов тоже приводил.
У хорошего адвоката (и хорошего врача) не бывает "одноразовых". Если специалист кидает клиентов - их очень скоро не останется и он останется без денег. (Видел много раз.)
Не вопрос. Но вреда от плохого слесаря несравнимо меньше, чем от плохого врача. Исправить работу самого плохого слесаря без особого труда сможет слесарь хороший. Чего не скажешь о врачах. Оттого и ответственность у них разная должна быть. Не должно быть плохих врачей! Это утопия, понятно. Но когда плохих становится больше хороших - вовсе кошмар.
Хм. Попробуйте лечиться у слесаря. 😛
Кстати, ответственность у врача и так вполне серьезная. Если доктор грубо накосячил или косячит постоянно по мелкому - никто его прикрывать не будет. Нахрена администрации клиники геморрой? (Исключения видел редко, только в случае очень хороших связей у накосячевшего. Хотя, все зависит от ситуации. Пару лет назад в одной престижной больнице москвы работал зав. отделением один армянин. Устроился, как водится, "по звонку" + 250 тыс долларов взятка. Работал, вроде, неплохо, но имел хобби - домогаться пациенток на грани изнасилования. После 2-й жалобы в администрацию - вылетел на улицу со свистом и с компенсацией потерпевшей морального вреда (чтобы заявление в милицию не попало).)
Проблема плохих докторов решается просто: обращайтесь к хорошим.
Проблема пищевых отравлений некачественными продуктами питания, решается покупкой заранее качественных продуктов. Ну-ну.
Так не надо на помойке питаться. И все хорошо будет. 😛
В серьезных делах - да. А в мелочи, вроде удаления вросшего ногтя - почему бы и нет?
А в медицине несерьезных дел не бывает. У любой манипуляции возможны осложнения вне зависимости от качества выполнения. Массу случаев знаю, когда простейшая ситуация печально заканчивалась. Помню, дочь главврача роддома рожала (молодая, здоровая). Тут уж качество оказания помощи, сами понимаете, было ого-го. Эмболия околоплодными водами и смерть в итоге. Или, еще в институте, на ортопедии: преподаватель "на полчасика" в операционную с занятия сходил. Грыжу диска удалить. Оперировали тремя бригадами 14 часов. (Вместо грыжи оказалась опухоль с повреждением брюшной аорты. Пациент выжил только потому, что рядом оказалась бригада сосудистых хирургов и у них был протез аорты под рукой.) Так что, 100% гарантии в медицине нет и быть не может. А уж определить где нормальный риск а где халатность - это экспертиза делает.
Ни чуть не лучше фразы вроде: "Медицина - полунаука, полуискусство".
А что? Так оно и есть. К точным наукам медицину отнести нельзя - слишком уж человеческий организм сложен. Да и у болезней слишком много причин бывает и все учитывать надо.

А хорошего врача найти просто: хороший - тот который результат обеспечивает. Если врача вам знакомые- родственники рекомендуют, то, после проверки на собственном опыте, можно зачислить доктора в "хорошие" и дальше только с ним дело иметь. Я, допустим, так стоматолога нашел и весьма доволен.

Dr. San

Doctor_D
Или, еще в институте, на ортопедии: преподаватель "на полчасика" в операционную с занятия сходил. Грыжу диска удалить. Оперировали тремя бригадами 14 часов. (Вместо грыжи оказалась опухоль с повреждением брюшной аорты. Пациент выжил только потому, что рядом оказалась бригада сосудистых хирургов и у них был протез аорты под рукой.) Так что, 100% гарантии в медицине нет и быть не может. А уж определить где нормальный риск а где халатность - это экспертиза делает.

Это, кстати, следствие некачественной диагностики.
Но ТОГДА, когда единственное доступное исследование - рентгенография, это было допустимо.
Ибо врач был не виноват, что не разглядел опухоли.

А сейчас, когда пациента просто запихивают в томограф и нате вам - полная картинка, это было бы уже преступной халатностью.

Так и в обсуждаемом случае.
Если начинается отслойка плаценты - то это сопровождаестся активизацией родовой деятельности. И изменением ЧСС плода.
Но в данном случае, похоже, плод и так был в гипоксии, так что отслойка не отразилась на его ЧСС, да и родовая деятельность не активизировалась... ("Ни что не предвещало...", так сказать)
И это не зависит от заплаченной суммы.

Но ВОТ ЕСЛИ начавшуюся родовую деятельность НАМЕРЕННО подавили медикаментами, чтобы роженица "долежала до понедельника" - тогда это преступная халатность, приведшая к гибели ребенка. И за это надо судить...

Mahaon007

У хорошего адвоката (и хорошего врача) не бывает "одноразовых". Если специалист кидает клиентов - их очень скоро не останется и он останется без денег. (Видел много раз.)
У "хорошего" адвоката, достаточно серьезных, и "неодноразовых" клиентов. Которым он и обеспечивает качественную услугу. А нищеброд, с трудом собравший на оплату, мало интересен. Он никогда не станет "многоразовым" в силу бедности и вообщем то законопослушности. А его деньги допустимо не разбивать на разные части. Самого же его, вынудить ещё оплатить дорогостоящие экспертизы и прочее. Результаты которых, останутся ему, как напоминание о его неудачливости.
Хм. Попробуйте лечиться у слесаря.
Основная масса россиян так и делает. Пользуясь услугами "слесарей" в районных поликлинниках и больницах.
Пару лет назад в одной престижной больнице москвы работал зав. отделением один армянин. Устроился, как водится, "по звонку" + 250 тыс долларов взятка. Работал, вроде, неплохо,
Как свинья устроившаяся "по звонку" может работать неплохо???
Сумма взятки хорошо демонстрирует, сколько собирался брать в дальнейшем.
Как собирался отбивать вложенные четверть ляма.
Как работать хотел, сволочь.
Так не надо на помойке питаться. И все хорошо будет.
А вы там иногда питетесь/лись? 😊 Я просто помойки по назначению использую. Мусор там выкинуть, ещё что ни будь. А еду покупаю в продуктовых магазинах.
Причем класса "премиум". Рынки, из за брезгливости и нац. взглядов - не посещаю. Равно, как всякие сомнительные палатки и павильончики.
А купить дрянь можно и там. Последний пример. Новогодние праздники. В ведро полетела банка кр. икры и кусок семги. Семга была с душком, а икра вовсе непонятного качества. Хорошо сразу заметили, а могли потравиться.
Это к тому, что не всегда качество можно проверить ДО, а не ПОСЛЕ события.
Да, сроки годности, и прочая информация сомнений не вызывала. Так же, как и дипломы и складная болтовня отдельных "специалистов".
А в медицине несерьезных дел не бывает. У любой манипуляции возможны осложнения вне зависимости от качества выполнения.
Всё, что связано со здоровьем - серьезно. Но степень серьезности разная.
Дать гарантию на результат шунтирования, или на результат лечения ОРЗ.
Есть разница?
И если с одной проблемой любой человек может справиться самостоятельно, в другой потребуются некие знания и опыт. А в третьей не обойтись без специалистов и аппаратуры.
А что? Так оно и есть. К точным наукам медицину отнести нельзя - слишком уж человеческий организм сложен.
Нет, не так. К точным наукам медицину никто и не относит. В контексте поликлинник, больниц и прочего - это ремесло. И ремесло надо делать хорошо.
Быть внимательным, знать, что ты делаешь, и принимать нестандартные решения по профилю в случае "вдруг". Не более.
А сократовских умозаключений или юнговских глубин анализа от участковых врачей и дежурных гинекологов никто и не требует. И открытий, космической ценности новых не ждут. Делай свое дело, и делай его хорошо. Всего то.

Doctor_D

Это, кстати, следствие некачественной диагностики.
Но ТОГДА, когда единственное доступное исследование - рентгенография, это было допустимо.
Не, там КТ делали. Грыжа как грыжа. Опухоль небольшая была, с инфильтрирующим ростом в стенку аорты.

Dr. San

Doctor_D
Не, там КТ делали. Грыжа как грыжа. Опухоль небольшая была, с инфильтрирующим ростом в стенку аорты.

КТ - это не ЯМРТ... С высоким разрешением...

Dr. San

Mahaon007
Как свинья устроившаяся "по звонку" может работать неплохо???

Как ни странно - может.
Случай курьезный, но тем не менее - БЫЛЬ.

Сразу после ординатуры работал в городской поликлинике неврологом.
Там же работал участковый терапевт. Хорошо работал, ни одной жалобы от пациентов, наоборот все были довольны. И так 3 года...
Пока всерьез не накосячил. Тогда при разборе полетов и выяснилось, что у него ВООБЩЕ диплома нет. 😀

А так и работал. В простых случаях (ОРЗ, давление) - сам назначал лечение. В более сложных - "советовался с коллегами".
Так бы и продолжалось... Но поставили его дежурить в субботу, когда "посоветоваться" не с кем было...

Кстати, никто так и не понял, как он на работу устроился. Тогдашняя зам по кадрам на пенсию ушла, а новая понятия о ситуации не имела...

Но главное - он со всеми бабульками-дедульками, которые на прием приходили (а их в поликлиниках - до 85% на приеме) - "муси-пуси". Бабульки довольны "внимательным отношением" "доктора". Начальство довольно отсутствием жалоб...

Doctor_D

Как свинья устроившаяся "по звонку" может работать неплохо???
Бывает и такое, иногда. Есть больницы, куда иначе чем за взятку или по блату и не устроишся в принципе.
Сумма взятки хорошо демонстрирует, сколько собирался брать в дальнейшем.
Как собирался отбивать вложенные четверть ляма.
Ну, дык. Он их и отбил скорее всего за полгода работы.
Дать гарантию на результат шунтирования, или на результат лечения ОРЗ.
И то и другое - вещи несложные. Но, умирают изредка в обоих случаях.
Делай свое дело, и делай его хорошо.
Ну, осталось только, чтобы заключение о вине врача делал специалист в медицине, а не в слесарном деле и все будет ОК. 😛 А то, если за каждого умершего пациента будут нанимать по киллеру - лечиться у слесарей придется.
КТ - это не ЯМРТ... С высоким разрешением...
Ну, коллега... Грешно смеяться... Середина 90-х, областной центр, обычная городская больница... Там, примерно в те же времена полгода даже без рентгена работали. В 96-м, помню, в областной больнице пациентов на плановые операции просили со своими наркотиками приходить...

Mahaon007

Doctor_D
[QUOTE][B]Ну, дык. Он их и отбил скорее всего за полгода работы.
Бедный, бедный врач... Особо бедствуют, как я понял армяне и прочие ЛКН в белых халатах. Ужос. Необходимо молоко давать за вредность и надбавку повысить. Всем зав. отделениям. Младшему персоналу - только молоко.
Ну а тому, кто у того армянина лавэ брал за прием - вообще премировать поездкой в санаторий и назначить льготу 50% оплаты жилья. Ибо сирым и убогим помогать надоть.
И то и другое - вещи несложные. Но, умирают изредка в обоих случаях.
Ни разу не слышал, что бы мерли от ОРЗ. От осложнений на сердце, на ещё что то вызванное ОРЗ - да. Но чисто от ОРЗ...
Какой хитрый недуг однако. 😀
В 96-м, помню, в областной больнице пациентов на плановые операции просили со своими наркотиками приходить...
Подтверждаю.
Одна мадам тады ходила на аборт со своим "пузырьком".
Там то ли не было, толи ещё что. Непомню уж. 😊

Dr. San

Doctor_D
Ну, коллега... Грешно смеяться... Середина 90-х, областной центр, обычная городская больница... Там, примерно в те же времена полгода даже без рентгена работали.

А я и не смеюсь 😊
Я о том, что при доступности ЯМРТ высокого разрешения данная операция была бы преступной халатностью. Сейчас.

А тогда, с КТ начала 80-х выпуска... Отнюдь...

Вот сейчас я работаю в обычной ЦРБ.
В 09-м нам поставят КТ последнего поколения. В 10-м - ЯМРТ последнего поколения. (Сейчас у нас ЯМРТ 94 года...) Вот тогда... 😛

Doctor_D

Бедный, бедный врач... Особо бедствуют, как я понял армяне и прочие ЛКН в белых халатах. Ужос. Необходимо молоко давать за вредность и надбавку повысить. Всем зав. отделениям. Младшему персоналу - только молоко.
Ну а тому, кто у того армянина лавэ брал за прием - вообще премировать поездкой в санаторий и назначить льготу 50% оплаты жилья. Ибо сирым и убогим помогать надоть.
Ну, на самом деле, таких хлебных мест не так много. Мой однокурсник проходил там ординатуру, случайно получил в палату старичка- пенсионера. На следующий день приехал племянник дедушки (на трех машинах с мигалками) и не глядя достал из бумажника сантиметр американских денег - чисто для познакомиться с доктором. "Если вылечите" - обещал еще больше + другие бонусы. 😊 Долечивал дедушку уже зав. отделением, разумеется. 😊
Наши армянские друзья 😊, конечно, очень стремятся на такую работу. Однако, отбор достаточно грамотных существует - клиентура серьезная бывает, да и клиника в мире известная, скандалы не нужны. Совсем уж никудышнего пристроят на бумажную работу, подальше от больных - сам уволится. 😛
Ни разу не слышал, что бы мерли от ОРЗ. От осложнений на сердце, на ещё что то вызванное ОРЗ - да. Но чисто от ОРЗ...
Какой хитрый недуг однако.
Навскидку, пару случаев:
Женщина, лет 40, зам главного бухгалтера банка. Заболела гриппом. Тоесть - типичная картина ОРЗ, небольшая температура, головная боль, плохое самочувствие, немножко кашель-насморк. В поликлинике выписали обычное лечение: витамины, панадол...
А это, блин, не грипп был, а начало острого гепатита Б. (По обычным анализам первые несколько дней определить нельзя.) Острый геп Б + панадол (не как доктор прописал, а побольше - чего эти доктора понимают вообще!) = фулминантная (молнеиносная) форма болезни. Труп через 3 дня. На вскрытии патологоанатом удивлялся - от печени практически ничего не осталось.

Женщина, около 50 лет. Лежала у нас в отделении с холециститом. Вылечили. Потом, перед выпиской - чисто ОРЗ (тем более - осень, эпидемия гриппа). Начали лечить. А это вовсе не ОРЗ а начало острого лейкоза. Исход летальный.

Mahaon007

Таки ви хотите сказать, шо это усё было ОРЗ? 😊

А так.. Диагностика никакая.
Такая же ответственность.
Равнодушие и безалаберность.

Doctor_D

А так.. Диагностика никакая.
Такая же ответственность.
Равнодушие и безалаберность.
Во-во. Вы так хорошо разбираетесь в медицине что выдаете такие оценки?

В обоих случаях - внешняя картина заболевания идентична ОРЗ.
Диагностика:
В первом случае: даже и не знаю. Вирусология делается 5- 7 дней (за это время обычно выздоравливают). ПЦР - дорогостоящий метод, невозможно назначать всем подряд. В том случае - в городе вообще не делали этот анализ из-за отсутствия лаборатории.

Во втором случае - изменения в анализе крови были сильно искажены быстро развившейся пневмонией (возможное осложнение для ОРЗ). Более детального обследования не было из-за быстрого развития болезни (больная умерла в реанимации на третьи сутки).


Mahaon007

Во-во. Вы так хорошо разбираетесь в медицине что выдаете такие оценки?
Разбираюсь. На достаточном для себя уровне, и недостаточном, что бы работать по этому направлению с другими официально. Хотя в свое время неоднократные предложения были. Пусть и не за 250 тыр. зелени, а бесплатно.
Не моё это, да и совесть не позволит.

В обоих случая приведены примеры летальных исходов лишь косвенно связанных с острым респираторным заболеванием. Разговор же шел о прямых случаях по этому диагнозу. Их нет. Здоровый человек не может загнуться от такого заболевания. Только если ОРЗ явилось катализатором других процессов, или диагноз был установлен неверно.

Примеры же из моей жизни, есть гораздо проще и понятней, чем неудачный поиск болячек неясного генеза.
Банально не услышали, не увидели, не выяснили, забыли про аллергика.
Не рассчитали дозировку, просто вовремя не поставили капельницу, что послужило прогрессии заболевания.
Случаев достаточно.
А обижаться, и отстаивать честь мундира - дело непростое.
Если Вы лично, хороший доктор, можете самому себе это сказать честно. И Ваши бывшие пациенты Вам это говорили - честь Вам и хвала. Удачи, и всяческих благ по жизни.
А нет - не в интернете же оправдание себе искать..
В любом случае, история с роженицей - крайне неприятная. И повторюсь, почему то кажется, она ещё не закончена.


Но мы отдалились от темы. Офф.

Doctor_D

В обоих случая приведены примеры летальных исходов лишь косвенно связанных с острым респираторным заболеванием. Разговор же шел о прямых случаях по этому диагнозу.
На момент установления первичного диагноза - все данные были за ОРЗ. Теперь представьте, если врач "дал гарантию". Виновен ли он? Стоит ли его "заказывать"?
Здоровый человек не может загнуться от такого заболевания.
А! Ну, менингококковая инфекция тоже в 80% случаев в виде ОРЗ (назофарингит) протекает. Да и чисто вирусные менингиты и энцефалиты бывают...
Примеры же из моей жизни, есть гораздо проще и понятней
Так и в моей достаточно. И, заметим, в большинстве случаев все решалось в рамках закона и здравого смысла.
Если Вы лично, хороший доктор
Да, это так. 😛
В любом случае, история с роженицей - крайне неприятная. И повторюсь, почему то кажется, она ещё не закончена.
Только теперь все осложнилось. Мужика закроют - 99%. А жене придется тратить деньги на передачи в тюрьму, а не на выяснение была ли вина врача. 😞

Mahaon007

Если Вы лично, хороший доктор


Да, это так.

Вот это самое главное. 😊
Важнее и моих, и всех остальных постов здесь.
На самом деле. 😛

PaulBaumer


А вот такой вопросец:
Я смотрю пациента, у которого шанс выжить процентов 10. Я могу попытаться ему помочь, но это очень трудо - и морально затратно. Я буду работать изо всех сил и знаний, но пациент помрет.
Хороший я доктор или плохой?
Морально ли отказаться от помощи этому пациенту с формулировкой "результат не гарантирован, а геморрой (жалоба) возможен?

Хотел бы обсудить именно моральный аспект, предполагая ситуацию что, юридически я волен поступать как считаю нужным.

vasia2009

пральная самка должна уметь и мочь рожать самостоятельно... такие в принцыпе ышшо есть если поискать... бо полностью разрушыть систему самовопроизводства юзеров (без зависимости от) таки неочень лехко - часть всеравно могет...

медыцына достаточно слабо влияет на здоровье юзеров... попытался спросить - какая фефективность медсистемы в процентах - ответили - в общебольницу поступают два типа юзеров - лехкие и тяжелые... так вот лехкие в основном сами выздоравливают и больница всево лишь мешает им самоуничтожаццо во время выздоровления (типа бухать и напрягаццо)... а тяжелых в 90 процентов отвозят в морг (причем некоторых ышшо недохлых, бо ждать пока они сами в теплом месте - бессмыслено... чиста как в теме - дохтур сказал - в морг - значит в морг), т.е. силами медицыны получаеццо вылечить ок 10 процентов всево - и это типа уже польза...

соотв в связи с фиговой фефективностью медицыны главная заповедь у дохтуров - не навредить... бо иначе юзер из самовыздоравливающево лехково могет стать тяжелым и ой...

>Я смотрю пациента, у которого шанс выжить процентов 10.

обычный тяжелый случай... если в очереди на осмотр сидят ышшо неск десятков леких юзеров - текущево отправить в морг... т.е. смысл ф чем - дохтуры не должны сильно мешать прынцыпу естесново отбора...

>Я могу попытаться ему помочь, но это очень трудо - и морально затратно. Я буду работать изо всех сил и знаний, но пациент помрет.
Хороший я доктор или плохой?

если за ето время можна было теми силами и знаниями помочь не стать тяжелыми нескольким лехким юзерам - плохой бо в результате экшена удалось с вероятностью 10 процентов оставить в жывых всево одну голову юзера и при етом стало тяжелыми и дохлыми больше одново юзера (т.к. из тяжелых действительно выжывают тока ок 10 процентов)...

если в больнице больше ваще никово небыло - ну можна и на етово тяжелово все силы тратить - больше некуда же... а во время экшена - набереццо экспы, шо есть полезно...

spec

vasia2009
пральная самка должна уметь и мочь рожать самостоятельно... такие в принцыпе ышшо есть если поискать... бо полностью разрушыть систему самовопроизводства юзеров (без зависимости от) таки неочень лехко - часть всеравно могет...

медыцына достаточно слабо влияет на здоровье юзеров... попытался спросить - какая фефективность медсистемы в процентах - ответили - в общебольницу поступают два типа юзеров - лехкие и тяжелые... так вот лехкие в основном сами выздоравливают и больница всево лишь мешает им самоуничтожаццо во время выздоровления (типа бухать и напрягаццо)... а тяжелых в 90 процентов отвозят в морг (причем некоторых ышшо недохлых, бо ждать пока они сами в теплом месте - бессмыслено... чиста как в теме - дохтур сказал - в морг - значит в морг), т.е. силами медицыны получаеццо вылечить ок 10 процентов всево - и это типа уже польза...

соотв в связи с фиговой фефективностью медицыны главная заповедь у дохтуров - не навредить... бо иначе юзер из самовыздоравливающево лехково могет стать тяжелым и ой...

Эта теория легко опровергается сведениями по странам и регионам, в которых медицина слабо развита.
В этом случае средняя продолжительность жизни крайне низка (может быть 25-30 лет например), практически все имеют букет различных заболеваний.
Ну про легких-тяжелых и про отправление живых больных в морг, думаю, врачи выскажутся лучше меня 😊

vasia2009

>сведениями по странам и регионам, в которых медицина слабо развита.

у них кроме медицыны (типа помогающей выжыванию) есть еще непомогающие выжываниию проблемы с питанием и жыльем и у некоторых - со здоровым образом жызни...

у развитой платной медицыны (как у большой бизнес системы) есть гадосное свойство - ей для работы и заработка нужны больные... и иногда она даже делает их из относительно здоровых...

причем соотв выгодны лехкие больные - на них нинада тратить многа средств, они сами выздоравливают, ими хорошо обеспечивать оборот пациентов, наполнение лечебных площадей, отчитываццо за бюджет и др...

а вот тяжелые невыгодны - на них очень сложно показать полезную фефективность от фсей отрасли...

соотв поэтому же - тратить силы на лечение тяжелово если есть очередь из лехких - невыгодно... выгодно быстро переработать лехких и по возможности не дать им скатиться до тяжково...

Добрыня Никитич

vasia2009
пральная самка должна уметь и мочь рожать самостоятельно... такие в принцыпе ышшо есть если поискать... бо полностью разрушыть систему самовопроизводства юзеров (без зависимости от) таки неочень лехко - часть всеравно могет...

С трудом - но всё же понял - Ваш язык, чем-то напоминающий русский. 😛
Что в Вашей терминологии означает "пральная (правильная?) самка"? По каким критериям Вы определяете правильность или неправильность "самок"?

Mahaon007

Добрыня, это вероятно, как и "пральный самец" не пользуясь наркозом, удаляет себе аппендицит не выпуская из мозолистых пальцев беломорины. Чуть распахнув промасленный бушлат. 😊
"Пральная самка" настолько же сурова. Рожает в поле, приносит в подоле, кормит "соской" из жеваного хлеба в тряпке. 😊
Ибо хлеб всему голова, а водка лекарство от всех недугов. 😊

Под эту классификацию кстати, как никто другой подходят самки мохноногих пришельцев.
Вот уж кто не озабочен выбором доктора, и чистотой трусов. Ибо трусы стирать, как минимум мыло "нуно", а мыло - хитрая и злая химия.
(А-а, самцы же их трусы не носят! Забыл совсем. Вера не велит.)
Да и рожают их самки, как говорится "не вынимая" 😀
Критерии "пральности" - сходятся.
Естество - наше "фсё"! 😀
Как то так. 😊

десант

уважаемы медики, обьясните мне тогда, почему по стране такой рост кесаревых?
что все роженицы в с отягощающими факторами идут?
почему прививки делаются с началом эпидемии, а не по срокам при которых иммунитет возникает?
почему детям делают прививки без ведома родителей и без обследований?
почему вы план по таким мероприятиям гоните?
почему назначаются антибиотики без посевов?в отделениях гнойной хирургии московских больниц.
и делать начинают только после твоих умных вопросов.
у меня ощущение, что вы как милиция работаете для "галочки" по показателям, а итог -здоровье людей вас совершенно не интересует.
план мероприятий спущеных сверху выполнили и ладно.

Mahaon007

Правильное у вас ощущение. 😞
Кесарят потому, что так проще. При малейшей непонятке начинают склонять к нему. Про антибиотики, аллергиков - то же самое. Никому морочится неохота. Квалификация большинства врачей сейчас - ниже плинтуса. В учебные заведения в основном - "ходят". Да и то.. Отношение "шуриков" к больному - материал с которого можно срубить бабла. Или нельзя. Но людей в них большинство из "дохтуров" не видят.
Очень много лкн -ов. Эти так же пришли "мыть бабло", а местных но "небогатых или жадных" пациентов просто ненавидят.
Для своих же знакомых сделают все, или будут являться для них "связующим звеном" с действительно специалистами выше.
Говорю это не только, как очевидец неоднократных событий, но и как знающий и имеющий знакомых медиков. Как старого поколения, так и сравнительно молодого. От профессуры до фельдшера скоряги.
И те, и те в приватных беседах, не скрывая рассказывают истинное положение дел. Да и ясновидящим тут быть не нужно.

P.S./
Сейчас снова будут разговоры о трудности жизни врачей, "низких доходах" и предвзятом отношении обывателей.
Повторяю - к хорошим докторам мои сообщения не относятся.
Но как же вас мало, хорошие!

DoctorD

уважаемы медики, обьясните мне тогда, почему по стране такой рост кесаревых?
Дык, как правило, сами тетки и просят - чтобы фигуру не испортить и в родах не мучиться. Чик - и готово. Доплатят доктору - и показания к операции найдутся.

что все роженицы в с отягощающими факторами идут?
Лучше диагностировать стали просто.
почему прививки делаются с началом эпидемии, а не по срокам при которых иммунитет возникает?
В принципе, можно сделать в любое время. А от гриппозной вакцины иммунитет быстро развивается. Чем ближе к эпидемии - тем сильнее будет (он не вечный).
почему детям делают прививки без ведома родителей и без обследований?
Потомучто идиеты. За такое элементарно к ответственности привлечь. (Знаю массу случаев.)
почему вы план по таким мероприятиям гоните?
А это система у нас такая. 😞 Помню, у моей бывшей в больнице (она врач-инфекционист) пришел "план по инфекционным заболеваниям". Доктора ходили и репу чесали: "это на что нас толкают? Сальмонеллу по городу разбрызгивать, что ли?" 😛)
почему назначаются антибиотики без посевов?в отделениях гнойной хирургии московских больниц.
Потому что посев будет готов дня через 3. Если больного это время не лечить - сгниет нахрен. Вот и используют проверенные комбинации препаратов.
итог -здоровье людей вас совершенно не интересует.
план мероприятий спущеных сверху выполнили и ладно.
Не так. Стараемся и больным помочь и идиотские указания мудрых администраторов выполнить (с наименьшим ущербом).
Но как же вас мало, хорошие!
Хороших - большинство, как не странно. (Сам удивляюсь!) Однако, уродов косоруких тоже хватает. А видно их гораздо лучше, к несчастью.
Поэтому: к врачу только по рекомендации знакомых или родственников. В крайнем случае - сначала пообщаться с доктором, потом уже решать - иметь с ним дело или нет.

shin-ap

Десант! Вы глас народа. В четверг был на корпоративном банкете. Сказал тост на тему, что сегодня мы врачи, завтра - обычные пациенты. Пожелал всем молодым специалистам побывать маленько пациентами 😊. А ещё напомнил, что все врачи умирают не от старости, а от профессиональных заболеваний 😞. Настоящие врачи.
Когда меня спрашивают, что бы я сделал в подобном случае, то ответ один: выезд за город и закапывание в землю живьём.

Dr. San

Mahaon007
Сейчас снова будут разговоры о трудности жизни врачей, "низких доходах" и предвзятом отношении обывателей.

Да не будет этих разговоров...
Все уже переговорено сотни раз...

Сейчас средняя зарплата врача в МО - 20-25 тыр.
Если смотреть "в среднем по России" - вроде и неплохо.
Но если смотреть с позиции предъявляемых требований, в том числе и населения, уровня ответственности, нервно-психической и физической нагрузки - должно быть на порядок больше.

Вот тут сравнивали врача со слесарем-сантехником. Слесарь-сантехник в МО зарабатывает те же деньги, 20-25 тыр (+-5 тыр). Но согласитесь, что сравнивать работу врача и слесаря "немного" некорректно.

Будет зарплата врача 200-250 тыр - вот и спрашивайте тогда "качество работы"... И исками грозите, и киллеров нанимайте...

Средняя продолжительность жизни хорошего хирурга - 50 лет. Плохого - 70...
Согласитесь, что требовать от врачей ХОРОШЕЙ работы при зарплате на уровне слесаря-сантехника - как-то "не очень"... Вот и уходят из медицины люди... Или "компенсируют" дельту взятками. Но лучше от взятки врач работать не будет. Не надейтесь. И от повышения зарплаты на 10-20% тоже.
И имейте ввиду, что эти 20-25 тыр - работа на 2 ставки. Т.е. двойная нагрузка за то же время. Соответственно, вдвое меньше времени на пациента и т.д...

Только адекватная оплата и жесткая связь оплаты труда с качеством работы...

Сколько "зарабатывает" депутат ГД, от которого зависит жизнь граждан страны?
Вот хотя бы столько и должен зарабатывать врач, от которого тоже зависит жизнь граждан страны. Даже если депутат никакой ответственности за "жизнь граждан страны" реально не несет... В отличие от врача...

"Верховным" на все это пох? Так сами же выбирали. "Голосовали сердцем". Расхлебывайте.

Вот в моем районе укомплектованность врачами - 50% (т.е. не хватает ровно половины), а из тех 50%, кто работает - 40% - пенсионеры, и еще 30-40% - "кто попало", кого нигде больше не берут, но "работать все равно некому"...
Почему?
Элементарно. Нет жилья ни для молодых специалистов, ни для приезжих.
Одно время пытались давать муниципальное. Но более 80% получивших жилье через год-два уходили на более высокую зарплату либо в Москву, либо в частники. Либо вообще из практической медицины.
А с 2005 года единственный вариант врачу обзавестись жильем - купить.
(Цены на жилье и зарплату врача сами сравните... У нас цена метра - 3-4 тыс. уе.)

ЗЫ. Подытожу. Государству пох на здоровье населения. Передохнет своё - завезут мохноногих. Которые не столь притязательны к социальным благам. И плодятся как кошки. А мрут - так их никто и не считает... Как и кошек...
Вообще, России, для обслуживания газовых и нефтяных месторождений, добывающей промышленности требуется 15 миллионов граждан. Остальные 130 миллионов - лишние...
Отсюда и делайте выводы...

vasia2009

>По каким критериям Вы определяете правильность или неправильность "самок"?

непральная не могет сама (или с помощью немного обученово самца-мужа) родить жизнеспособное потомство в требуемом количестве (до 10..20 голов) без помощи каких-то врагов...

пошто тута хотят закопать дохтура я таки не понимаю - мужык сам отдал свою самку врагу...

>как никто другой подходят самки мохноногих пришельцев.

они почему-то практикуют жызнь вместо цывилизацыи... и достаточно результативно...

>что все роженицы в с отягощающими факторами идут?

их такими вырастили/вывели друзья народа...

>Будет зарплата врача 200-250 тыр - вот и спрашивайте тогда "качество работы"... Только адекватная оплата и жесткая связь оплаты труда с качеством работы...

китайцы платят врачам тока пока здоровые... если заболеют - врача пи3дят пока не вылечит... вроде вполне жеская связь оплаты с качеством лечения... мудрость какая-то в етом есть... у врача появляеццо повод сохранять здоровье юзеров максимально долго...

Dr. San

vasia2009
китайцы платят врачам тока пока здоровые... если заболеют - врача пи3дят пока не вылечит... вроде вполне жеская связь оплаты с качеством лечения... мудрость какая-то в етом есть... у врача появляеццо повод сохранять здоровье юзеров максимально долго...

У Вас слишком оптимистичное мнение по поводу Китайской медицины.

Да, в Китае есть нормальная медицина. Доступная единицам. С доходом не менее 200 тысяч долларов в год.

Остальное население довольствуется услугами "народной медицины". Самый доступный вариант которой для 70-80% населения - уличный иглоукалыватель, который, сидя в уличной пыли, "стерилизует" уже использованные иглы во рту (своём)...

vasia2009

остальной миллиард китайцев жывет назло врачам и почему-то не дохнет... как бы так и другим научиццо... тута ведь уже печатали - без медицыны никак низя выжыть - всево через неск десятков лет многа болезней и смерть...

>Доступная единицам. С доходом не менее 200 тысяч долларов в год.

ето фанаты лечения у медицыны - если они даже не выздоровеют - население особо не убавиццо...

>"стерилизует" уже использованные иглы во рту (своём)...

дык слюна юзера вполне так йад...

vasia2009

>почему назначаются антибиотики без посевов?

нуу есть оказываеццо такой простой и дешевый способ проверить - юзера кормят и поят неск суток обычной едой - если не выздоровел - значиццо бактериальное и надо ему поесть антибиотиков...

мя так недавно лечила медицына - я сначала даж не понял пачиму при описаных в мануале сложных и обязательных анализах медицына просто сразу назначила поесть антибиотиков... оказываеццо я уже с момента заражения прожыл неск суток и если было бы не бактериальное - выздоровел бы уже...

Dr. San

Васенька, тупой стеб без аргументов - отстой...

pepper70

Из моего выпуска 94 года в медицине осталось % 40.
и не лучших...

------------------
Грех предаваться унынию, когда есть другие грехи!

десант

по кесареву.
встречал таких теток, которые хотят сами операцию. правда в основном обьясняли страхом родов, а не желанием сберечь фигуру.
но к сожалению, сейчас все больше кесарят принудительно.
диагностика здесь не причем, читал интенрвью главного врача страны(должность точно не помню),так он очень сильно возмущался подобной тенденцией и называл ее неоправданной.
моей жене сделали кесарево, хотя до родов ни каких показаний к этому не было вообще.
после родов все врачи с которыми сталкивались, выражали свое удивление "а зачем кесарили".
знакомая не может найти роддом в рязане, чтоб нормально родить.
кого не спросишь из молодых рожениц -все через кесарево.

Hail

Господа, вы в институте учились? Помните, как там происходит учебный процесс, с зачетами, пересдачами и тд. Есть люди которые учатся хорошо и есть которые плохо, так вот в мединституте все точно так же. Кто учится хорошо, потом идут в ординатуру и становятся неплохими спецами и работают в тех мед учреждениях, где ценят их знания и навыки, получая за это соответствующие $, а кто не учился - те идут в поликлиники, в городские больницы и тд и сидят там за 6000 рублей. Если хотите снизить риск при родах, то платите 60000 - 100000 за контрактные роды в спец учреждениях, там сделают полное сопровождение и всю диагностику. Если вы любите рисковать или не хотите платить деньги то идите к недоучкам в жен. консультацию в поликлинике.
З.Ы. Никогда не приходите со своими детьми в городские поликлиники, именно там дети и заражаются от других людей, пока стоят в очереди.

десант

ерунду говорите.

vasia2009

>не приходите со своими детьми в городские поликлиники, именно там дети и заражаются от других людей, пока стоят в очереди.

немощные - заражаюццо и дохнут... пральные - заражаюццо и повышают иммунитет...

эт проста метод относительно пральных прививок... есть такой древний обычай - отправлять свое потомство к выздоравливающему от заразной болезни... при етом бытовая отмазка - типа сходить проведать таварищща... реально - сходить получить дозу заразы...

Dr. San

десант
ерунду говорите.

Вот это Вы зря.
Все правильно сказано.

В детской поликлинике должен быть (по нормативам) отдельный вход для больных детей, отдельный - для здоровых.
И те и другие НЕ ДОЛЖНЫ пересекаться в общем коридоре. Контакт больных детей со здоровыми вообще должен быть исключен...

десант

в селах вообще нет детских поликлиник, приходится ходитьв сельские амбулотории и дня рбенека то же нет.
вы судите по столице и очень крупным городам. а большинство населения страны живет совсем по другому.
и пожелания "Если хотите снизить риск при родах, то платите 60000 - 100000 за контрактные роды в спец учреждениях, там сделают полное сопровождение и всю диагностику. Если вы любите рисковать или не хотите платить деньги то идите к недоучкам в жен. консультацию в поликлинике"-это просто издевательство.
по принципу "за мкадом жизни нет".

DoctorD

"Если хотите снизить риск при родах, то платите 60000 - 100000 за контрактные роды в спец учреждениях, там сделают полное сопровождение и всю диагностику. Если вы любите рисковать или не хотите платить деньги то идите к недоучкам в жен. консультацию в поликлинике"-это просто издевательство.
по принципу "за мкадом жизни нет".
За МКАДом расценки дешевле. Кстати, лучше всего брать "страховой полис". Это немного дороже, чем платить напрямую больнице, зато в случае спорных ситуаций гарантирует защиту (страховая компания будет сама заниматься экспертизами и прочим).

десант

там и зарплаты ниже.
а еще не везде есть эти медучреждения со страховыми программами.
зачем вообще нужны тогда медики, если я для более благоприятного результата лечения(гарантий то все равно никто не даст, область не та)
тратить такие деньги.

vasia2009

>или не хотите платить деньги то идите к недоучкам в жен. консультацию в поликлинике

о как буржуины жывут - http://a-naumov.narod.ru/books.files/book1/m_tew.html ... роды в своем жылище под присмотром вмеру обученово юзера самые безопасные... а вот спецродклиники за аццкое бабло как-то самые смертельные получаюццо...


vasia2009

>зачем вообще нужны тогда медики,

шоб оказывать услуги с понтами... ну как же покупающие одежду в бутиках пойдут рожать в какой-то сарай общегражданский... да и для гарантированново оказания интимных услуг в законе - типа если аборт офицыально не хотят - так отправить самку в такую клинику хде оно гарантировано потомства не произведет жывово и фсе будет по закону и прикрыто актами и справками - нищасный случай, фсе боролись за жысь - и нихто лишний ниче не узнает... да и далее даже беременеть больше не смогет и будет в щщастье и праздности дожывать свой век...

дорогая медицына нужна для решения дорогих задач юзеров с большыми доходами... не могут же они решать их с помощью медицыны для нищщих - несообразно однака, да и утечки лишние пойти могут иль враги за копейки могут переломить ход течения болезни важново члена семьи...

а тута вот наблюдаюццо попытки перенаправления ааще всех юзеров в медицыну для юзеров с большыми доходами...

Andrew_M

Вот, к примеру выуженная из тырнета "заметка", о качестве и обычаях нашей медицины в городе столице.
Вначале огребаешь от уродов низачто, скопище коих на наших улицах не волнует тех кто обязан с ними бороться. А потом полуживой огребаешь порцию хамства и вымогательства от так называемых докторов из аула. Какое уж там проводится лечение, можно только догадываться. http://fatey.livejournal.com/21807.html

vasia2009

>Вначале огребаешь от уродов низачто, скопище коих на наших улицах не волнует тех кто обязан с ними бороться. А потом полуживой огребаешь порцию хамства и вымогательства от так называемых докторов из аула.

юзеров мяхко и на тормозах спускают к пральной самостоятельной жызни... по слухам могло быть фсе намного хужее...

DoctorD

десант
там и зарплаты ниже.
а еще не везде есть эти медучреждения со страховыми программами.
зачем вообще нужны тогда медики, если я для более благоприятного результата лечения(гарантий то все равно никто не даст, область не та)
тратить такие деньги.

И цены - соответственно. Зависит от города и уровня жизни в городе.
Страховые есть во всех более-менее крупных городах. Из мелких городов и из села лучше ездить в город - там медпомощь, как правило, качественнее (за редчайшими исключениями).
Зачем медики нужны? Странный вопрос. Когда мать меня рожала, у нее было осложнение в виде кровотечения. В роддоме ей спасли жизнь. Во внебольничных условиях - 100% смерть.

О гарантиях: апендицит - без лечения смертность 80%, с лечением -0,02% или типа того.

десант

у брата аппендецит (он курсант на тот момент), поступает в санчасть училища, а оттуда в окружной госпиталь.
в итоге коммисованный и без здоровья.
резали 4 раза. и это в областном центре.
ездить за медпомощью в ближайший крупный город -замечательный совет, только как вы себе эту процедуру представляете, сколько стоит такие поездки, где жить сопровождающим и так далее.
нормальная медецина не по карману большинству населения страны.

DoctorD

у брата аппендецит (он курсант на тот момент), поступает в санчасть училища, а оттуда в окружной госпиталь.
в итоге коммисованный и без здоровья.
резали 4 раза. и это в областном центре.
Бывает. Не все осложнения можно предусмотреть. Хотя, в данном случае - скорее всего, позднее обращение. У нас был случай, когда мама-педиатр (КМН) больше недели лечила сына лет 14-ти с типичным аппендицитом от "кишечной инфекции" дома. Как не странно - спасти ребенка удалось.
ездить за медпомощью в ближайший крупный город -замечательный совет, только как вы себе эту процедуру представляете, сколько стоит такие поездки, где жить сопровождающим и так далее.
Большинство может позволить. (Я жил в деревне, знаю.) Сопровождающим жить не нужно. Если только в качестве сиделки. Тогда живут прямо в больнице (при необходимости, больница с радостью согласится, т.к. младшего медперсонала не хватает хронически).
нормальная медецина не по карману большинству населения страны.
А это так, практически в любой стране. "Нормальная" медицина стоит дорого.
Для примера: для того, чтобы избавиться от очередей в поликлиниках надо увеличить число участковых терапевтов в несколько раз и увеличить норматив на одного больного хотябы до 20 мин (сейчас, кажется, 5-7, реально еще меньше). Для улучшения качества помощи необходимо увеличить число врачей-специалистов в поликлинике и улучшить лабораторно-диагностическую базу. Таким образом - возрастают расходы на зарплату (если мы хотим хороших врачей - им надо платить достаточно), надо построить несколько новых поликлиник, купить оборудование для них.
Я уже не говорю о расходах на нормальные лекарства.
Собственно, выходов два: либо страховка (дорого!) либо гос. финансирование (увеличение налогов - та же страховка но принудительная).
Скорее всего - будет определенная дифференцировка. Государство платит за малоимущих, богатые платят сами. (Разница в качестве естественно, будет. А куда деваться? Не могут все ездить на 600-х мерседесах.)
Так что - вкладывайте средства в профилактику и лечите болезни не запуская их. Я вот, недавно, запустил банальный кариес. В итоге, вместо 500 рублей выложу 7000. Вот такая арифметика.

десант

все это красиво в теории. в жизни получается, что с появлением одного стационарного больного, жизнь всей семьи рушится.
одни начинают ударно бабло для лечения зарабатывать, другие отправляются сиделками в больницу. хорошо, если для этого увольнятся не придется.
тот же транспорт найти большая проблема.

DoctorD

все это красиво в теории. в жизни получается, что с появлением одного стационарного больного, жизнь всей семьи рушится.
одни начинают ударно бабло для лечения зарабатывать, другие отправляются сиделками в больницу. хорошо, если для этого увольнятся не придется.
тот же транспорт найти большая проблема.
Увы. Система такая. За лечение так или иначе придется платить. Здесь вопрос только в минимизации расходов. По другому не получится. А вот в плане минимизации расходов многое сделать можно.

десант

понимаете, то что халявный сыр только в мышеловке я и так знаю.
только вот лично сталкивался с такой проблемой при обращении к платной медицине (говорю за себя и за жену)
1.назначение дорогого лекарства, по распространению которого у доктора и фармокологов договоренность -6 раз.
названия лекарства при этом пишется не на бланке рецепта, а на визитке врача, после требования рецепта почему то на бланке пишется нормальное лекарство. (это в разных учреждениях)
2.назначение обследованний, не имеющих ни какого отношения к обращению больного, без записи в медкарточку. после посещения главврача все назначается нормпльно. - 3 раза.
3. не назначение процедур и обследований из-за их дороговизны, хотя после обращения в страховую компанию все решается (я по полису ДМС оброщался)-2 раза.
4.случай с женой, который меня довел до каления.
в бюджетной поликлинике не смогли поставить окончательный диагноз из-за невозможности провести необходимые исследования. направили в микрохирургию глаза Федорова. там обратились как платные клиенты, все оплатили и пришли на прием к врачу, врачу показываем направление на обследование и предварительный диагноз. итог-обследование не назначает, диагназ ставит другой, назначает лечение и предлагает зайти через недлю, при этом открывает тумбочку и требует положить 300 рублей. карточка не заводится, рецепт на бумажке, а не на бланке.
иду к главврачу и пишу заявление -деньги вернули, карточку завели, обследование назначили, подтвердился первичный диагноз.
а если бы я жену лечил по методу этого урода, то что тогда?
я согласен заплатить деньги за лечение, но меня даже не пытаются лечить, а наооборот пытаются дольше больным держать, чтоб деньги доить.

DoctorD

1.назначение дорогого лекарства, по распространению которого у доктора и фармокологов договоренность -6 раз.
названия лекарства при этом пишется не на бланке рецепта, а на визитке врача, после требования рецепта почему то на бланке пишется нормальное лекарство. (это в разных учреждениях)
Другой вариант решения - спросить в аптеке или посмотреть в справочнике международное название и купить более дешевый аналог. Еще лучше - спросить у врача сколько стоит лекарство и есть ли адекватная замена дешевле. (Если врач дальше разводит - больше к нему не ходите.)
2.назначение обследованний, не имеющих ни какого отношения к обращению больного, без записи в медкарточку. после посещения главврача все назначается нормпльно. - 3 раза.
Здесь рулит ДМС. Страховая за такое порвет больницу как тузик грелку.
4.случай с женой
Абсолютно верные действия. На будущее - постарайтесь найти врача с хорошими рекомендациями.

десант

1.знакомые фармацевты то же жаловались, что их заставляют в аптеках продовать дорогие лекарства.
все ситуации в разных больницах, это уже система так работает.
2.это и было все по ДМС.
я специально описал случаи только в платных поликлиниках.
3.найти то постараемся. только тяжело подобрать врачей по всем специлизациям.

я все это привел, чтоб показать несостоятельность тезиса о платной медецине у нас, как способе избежать проблем с здоровьем.
и даже люди у которых полисы ДМС и есть деньги, могут запросто оказаться в бюджетной больнице в экстренном случаи.

DoctorD

3.найти то постараемся. только тяжело подобрать врачей по всем специлизациям.
А по всем и не надо. Нужен хороший терапевт (+ педиатр, если есть дети), стоматолог. Хирург - желательно (для мелких операций, травм). Остальные специалисты - по имеющимся хроническим болезням.

я все это привел, чтоб показать несостоятельность тезиса о платной медецине у нас, как способе избежать проблем с здоровьем.
*Избежать* - это, в большей степени, к самому человеку вопрос. Большинство болезней - от неправильного образа жизни.
А медицина полюбому платная. Вопрос только в том, кто именно платит. В конце концов, даже если платит государство - оно эти деньги из нашего кармана достает.

десант

про то и речь.
что нет ответственности у медиков (с этого все начиналось)
то есть деньги они по любому получают, а вот услуги оказываеют некачественные.

DoctorD

нет ответственности у медиков
Да как же нет-то? Наоборот, знаю немало случаев, когда ее чересчур много. (Типа исков "за больно сделанный укол".)
В остальных случаях - зависит от "потерпевшей стороны".

михон

десант
а вот услуги оказываеют некачественные.
Не услуги, а медицинскую помощь 😊 Но это так - мелкая придирка. На самом деле, врачи - такие же люди, как и все. И процент уродов, лентяев, раздолбаев, просто малограмотных среди них такой же высокий, как и среди других граждан. Поэтому здесь уж как повезёт. Но на самом деле хороших врачей у нас много. Нужно только знать, где их искать. Я лично хоть в обычную поликлинику, хоть в дорогущий частный центр никогда не пойду и родственников своих не отправлю.
Просто позвоню своим однокашникам, благо за прошедшие после выпуска годы все области медицины у нас охвачены.

DoctorD

Я лично хоть в обычную поликлинику, хоть в дорогущий частный центр никогда не пойду и родственников своих не отправлю.
Просто позвоню своим однокашникам, благо за прошедшие после выпуска годы все области медицины у нас охвачены.
+500. Или по рекомендации.
Уровень врачей в поликлиниках - ниже плинтуса, часто. У меня бывшая жена специальную тетрадочку вела, куда косяки поликлинических врачей записывала. Довольно много забавного, но, в основном - ужас, конечно.

михон

DoctorD
специальную тетрадочку вела, куда косяки поликлинических врачей записывала
Я одно время коллекционировал направления из поликлиник, с которыми ко мне люди приходили. Самое невинное: "Направлен на консультацию к ангелохирургу" 😀 По всей видимости всё-таки имели в виду меня, то есть "ангиохирурга"

Dr. San

"История N1.2 «Как мне на халяву вырвали зуб»

Что самое удивительное, как раз перед отъездом я основательно подлечил зубы в нашей платной и достаточно известной киевской стоматологической поликлинике. Не помогло. На третий месяц моего пребывания в США я почувствовал, что, столь искусно поставленная украинскими врачами пломба приказала долго жить. Вначале у меня еще теплилась надежда на то, что я смогу дотянуть до Киева, и там-то мне все сделают. Не дотянул. К тому времени у меня уже была дешевенькая зубная страховка (тут у них отдельные страховки на медицинское обслуживание и отдельные на зубы), и я решил ее использовать.

Записался на прием и стал ждать, потом бегать по стенам, а обратно по потолку.. больновато было. Наконец, я дождался. Прихожу в клинику, врач, естественно, афро-американка, кто же еще с моим-то счастьем. Посмотрела она на мой зуб с дыркой, заставила сделать рентген (я так понял, что они без рентгена тут даже от насморка не лечат) и вынесла вердикт - одна пломба 95 баксов. Такая сумма покрывалась страховкой, и я согласился. Тут я, конечно, должен склонить голову перед их аппаратурой, все у них продуманно идеально, все стерильно и одноразово. Единственное, что мне не понравилось, это склонность американских врачей делать очень сильные обезболивающие уколы. Я потом свои наблюдения сравнивал с тем, что рассказывали коллеги, так они обезболивающее дают даже при лечебном массаже. Короче, первый раз в жизни мне ставили пломбу под местным наркозом. Все сделали, и я довольный ушел домой. Кстати о наркозе, он еще на следующий день держался, в результате все на работе думали, что я с утра где-то поддал: язык все еще находился под анестезией и ворочал я им не очень.

Через три дня я уже не бегал по потолку, я там поселился! То ли она мне пломбу неправильно поставила, то ли высверлила неправильно, не знаю, но жить из-за еще худшей зубной боли просто не хотелось. Так как были, как раз суббота и воскресенье, я мужественно терпел, а в понедельник, ровно в 8 утра я уже заходил в клинику. Захожу, говорю (вежливо, блин, так говорю): Вы, госпожа хорошая, че-то не то сделали - болит, однако. Она меня в кресло усадила, посмотрела. Покачала головой, дала мне в руки рентген и направление к другому врачу, он, мол, вам поможет. Делать нечего, я через весь город к нему. Захожу, там сразу предлагают заплатить 50 баков аванса, с криком (про себя): «Подавитесь!», - плачу. Садят меня в кресло, и на этот раз уже мужик осматривает зуб. Он меня обнадеживает, что все будет хорошо. Я от боли уже тупею и начинаю говорить с ним на русском. И тут до меня доходит, что слишком подозрительные предметы мужик на стол выкладывает. Из последних сил интересуюсь, что это он задумал. Как что? Удивляется он, рвать будем. Нормально, думаю, а ведь хотел просто пломбу поставить:. Но честно, мне уже все равно было. Вкололи мне кубика три анестезии и вырвали. Отдам им должное, идеально вырвали, за две минуты, ломать - не строить.

Вот сижу я сейчас, смотрю на счет, который уже пришел за вырванный зуб и думаю. То ли хорошая у них медицина, что так быстро зуб вырвали, то ли плохая, потому как я, вообще-то, только пломбу хотел поставить. Не знаю.

Эпилог.

А потом мне позвонила врач, та, которая пломбу ставила и сказала, что зуб мне вырывали бесплатно и сумму с чека мне вернут, типа, их мне подарок. Нормальный подарочек, да? Вот, так и получилось, что зуб мне вырвали на халяву. Для особо слабонервных отмечу, зуб был зубом мудрости и никто при написании этой заметки, кроме самого автора не пострадал.

История 1.3 «Почему я никогда не буду болеть в Америке. »

Не буду рассказывать, как и при каких обстоятельствах это получилось, но факт остается фактом - я вывихнул руку. Причем конкретно вывихнул. Причем в 7 утра. Делать нечего, подумал я, придется обращаться к местным эскулапам. Я позвонил своему другу Олегу, мол, срочно приезжай, надо кореша спасать.

И вот мы поехали через весь город в больницу, где у меня принимали страховку. Приезжаем к входу для скорой помощи. Спрашиваем у полицейского на входе: где можно запарковать машину? Он говорит, мол, за углом - без проблем. Паркуем, я бочком ковыляю в приемное отделение больницы. Приемное отделение мне не понравилось сразу. Начиная с того, что мне буквально пришлось переступить через негра, который лежал с закатившимися глазами на полу в зале ожидания. Вокруг него суетились две медсестры, но ему, по-моему, было все в тот момент фиолетово.

В общем, подхожу я к стойке регистратуры, помогите, говорю, люди добрые. В ответ, с неизменной улыбкой, которую я ненавижу, мне дали листов 30 различных анкет и мило попросили заполнить. У меня рука правая вывихнутая, висит плетью. Я не то, что писать, сигарету закурить не могу. А они - ничем помочь не можем, как заполните - вылечим. На мое предложение, заполнить им самим под мою диктовку, они твердо ответили, что у них такие правила. Нет, ну слава богу, там Олег был, который все это заполнил, а если бы не было? Мдя.

В общем все было заполнено, меня посадили на каталку. Хотя с ногами у меня все было ок, у меня с рукой фигово было. Завезли в маленькую комнатку, ждите, говорят. Жду. А ведь больно, вывихнутая кость пережала вены, и рука потихоньку начинает неметь. Короче, доходить я начинаю потихоньку.

Через полчаса заходит медсестра. «Плохо?» - спрашивает. «Плохо», - отвечаю я. «Будем лечить», - уверяет она, измеряет мне температуру, записывает это все в бюллетень и уходит. Полчаса опять никого нет.

Еще через полчаса заходит медбрат. «Плохо?», - спрашивает. «Плохо», - отвечаю я. «Будем лечить», - уверяет он, измеряет мне давление, записывает это все в бюллетень и уходит. Полчаса опять никого нет.

Я начинаю дуреть и достаточно громко стонать. Через полчаса заходит еще одна медсестра. «Плохо?», - спрашивает. «ХРЕНОВО!», - отвечаю я. «Да Вы не волнуйтесь, вылечим!», - успокаивает она, и заставляет меня заполнить дурацкую анкету на отсутствие аллергенов на обезболивающее. Полчаса опять никого нет.

Короче, к тому моменту, когда ко мне все-таки пришел врач, прошло без малого два часа. Первое, что он мне сказал, это, мол, ты парень - счастливчик - с утра вывихнул, если бы в обед - часа четыре бы ждал. Знаете, если бы я не вывихнул правую руку, я бы ему выбил челюсть. Так что еще неизвестно кто из нас счастливчик. Дальше, он мне проникновенно рассказал какие у них продвинутые технологии наркоза, но при этом вскользь заметил, что в крайне редких случаях люди из этого наркоза не выходят по причине остановки сердца. И тут же дал подписать бумагу, что, если что, я к ним претензий иметь не буду. Я перекрестился и подписал, а что делать.

Дальше меня повезли на рентген. Там здоровенный негр попытался меня заставить встать у рентген аппарата и поднять руки на уровень плеч. Мол, это утвержденная позиция для снятия рентгена. Я ему, мол, ты идиот или кто? У меня рука вывихнута, не могу я ее поднять. Он уперся: правила, говорит, такие. Позвали врача. Он, правда, тот же вопрос насчет идиота негру задал. В общем, сделали мне рентген, вкололи какую-то гадость и вправили руку. Потом положили на каталку и вывезли в коридор, где и оставили. А наркоз у них прикольный. Явно дурь какая-то. Лежу я в коридоре и наблюдаю за посетителями. А в носу трубки торчат с кислородом, прямо как в кино. За этим увлекательным занятием меня Олег и застал. «Бери шинель, пошли домой», - говорит. А я ему: «Куда ж я пойду? Мне же еще гипс не наложили». Зову врача, спрашиваю: когда же меня в гипс закатывать будут? Врач так удивился: а зачем? Дали мне какую-то повязку марлевую на липучках, чтобы не приставал. Я в шоке. Когда я в Одессе руку вывихнул, меня по самое не хочу в гипс закатали, насколько я помню.

В общем, вышли мы из больницы. Олежка злой, потому что у него машину оштрафовали, а я довольный, потому как еще под наркозом. Кстати, насчет машины, сделаю маленькое отступление. Машину Олега оштрафовали за неправильную парковку, хотя нам на это место указал сам полицейский, который дежурил у больницы. У меня, кстати, такое подозрение, что он же и оштрафовал. Причем на 100 баксов оштрафовали. Мы обиделись. И пошли в суд. Записали, как зовут того полицейского. Сняли копии всех бумаг из больницы, что в то утро Олег вез меня в больницу, и это, буквально, было делом жизни и смерти. Нам казалось, что с такими весомыми аргументами восстановление доброго имени Олега - вопрос практически решенный.

И вот суд. Перед Олегом в суде выступала какая-то панковатая девчонка. Ее спрашивают: почему штраф не платите? Вы же были оштрафованы там-то и там-то, тогда-то. А она в наглую: а никаких таких квитанций я не видела, и вот свидетель есть - тыкая пальцем в здоровенного мексикоса уголовной наружности. Судья мекса спрашивает: она точно не видела? А мекс головой кивает, потому как явно вопроса не понял. Ну, судья молотком по столу - оправдана. Олежка духом воспрянул. Если эту прохвостку оправдали, то за такое правое дело как спасение товарища, наверняка, еще и медаль дадут. Нифига! Сказали, что пусть он хоть беременную жену вез бы на роды. Но ПДД святы и незыблемы. Заплатили мы эти 100 баксов на двоих и зареклись на правоблудие их уповать в дальнейшем.

Но это еще не конец. Когда Олег заполнял формы, он, видимо, где-то ошибся или америкосы в силу своего тупоумия не все поняли, но страховку мою, почему-то, не засчитали и прислали мне счет на полную сумму. Угадайте на какую. Нет, ну вы представьте просто. Мне измерили температуру, давление, вкололи обезболивающее и дернули за руку. Представили? Нет, не угадали. Еще больше. 1800 долларов. Тысячу восемьсот. В любом нашем травмпункте это делают бесплатно.

И уже, так сказать, в дополнение расскажу вам про их аптеки. В их аптеках в свободной продаже лекарств нет. Вернее нормальных лекарств. Там есть все, кроме нормальных лекарств. Там есть 20 видов аспирина и 15 видов колдрекса, 10 видов тампаксов и 5 видов капель в глаза от пыли. Антибиотиков там нет и никогда не было. Да что там антибиотиков, там супрастина даже нет (кларитин по-ихнему). Да черт с ним с супрастином, там даже аналог нашего бисептола и тот только по рецепту продается.

То есть схема такова: если вы даже знаете чем лечиться, вы все равно должны идти к врачу за рецептом. Визит к врачу, естественно, платный. Врач вам выписывает рецепт на 5-6 таблеток. Кончатся, говорит, придете еще. У меня так и было. Я за три дня 6 таблеток выпью и опять к врачу, мол, выпишите еще. Тот с удовольствием выписывает еще на три дня. Каждый визит платный. И врачам хорошо и аптекам.

А вы еще попробуйте доказать врачу, что вам эти таблетки нужны! У меня гайморит был страшенный. А мне в течение двух визитов врач говорила, что это у меня аллергия. На третий раз я ей возразил, что, мол, не может быть аллергии, когда гной идет. Я ее буквально попросил диплом показать о врачебном образовании или выписать мне, наконец-то, антибиотики. Выписала. Дура. Я из-за нее месяц мучался с ее аллергией. Антибиотики на третий день все симптомы сняли."
http://guns.allzip.org/topic/33/318389.html

Dr. San

михон
Не услуги, а медицинскую помощь 😊

Ошибаетесь.
Именно УСЛУГИ!!!

Вам, как клиницисту, не знать это простительно 😛. Но тем не менее это факт.

И ОМС оплачивает "оказанные услуги".
И даже при лицензировании медицинской деятельности лицензируется, например, оказание "услуг по ангиохирургии".

Посмотрите Приказ МЗСР N323 от 10 мая 2007 года...

CyberDaemon

finder00
годик отдохните от раздела, успокойтесь...


+1....

Doctor_D

"Направлен на консультацию к ангелохирургу"
😊 😊 😊

десант

михон
Не услуги, а медицинскую помощь 😊 Но это так - мелкая придирка. На самом деле, врачи - такие же люди, как и все. И процент уродов, лентяев, раздолбаев, просто малограмотных среди них такой же высокий, как и среди других граждан. Поэтому здесь уж как повезёт. Но на самом деле хороших врачей у нас много. Нужно только знать, где их искать. Я лично хоть в обычную поликлинику, хоть в дорогущий частный центр никогда не пойду и родственников своих не отправлю.
Просто позвоню своим однокашникам, благо за прошедшие после выпуска годы все области медицины у нас охвачены.
нравятся мне ваши советы.
а что делать человеку у которого все знакомые и друзья не имеют отношения к медецине?
а что делать человеку ,живущему на селе, где выбора то врачей вообще нет, а ближайший специалист с рекомендациями в 200 км.
что делать в командировке или в отпуске?
а уж "
На самом деле, врачи - такие же люди, как и все. И процент уродов, лентяев, раздолбаев, просто малограмотных среди них такой же высокий, как и среди других граждан. Поэтому здесь уж как повезёт.",
вообще классная фраза. она означает только одно -что система медецинского образования развалена, так как все медики дипломированные специалисты и с такими качествами при нормальном обучении никогда бы не получили диплома.
и тогда возникает снова вопрос -а зачем вся эта медецинская система нужна, если образование туфтовое, качество работ никакое, а разницы в в платной и бесплатной медецине не наблюдается.

десант

DoctorD
Да как же нет-то? Наоборот, знаю немало случаев, когда ее чересчур много. (Типа исков "за больно сделанный укол".)
В остальных случаях - зависит от "потерпевшей стороны".
это в нашей стране?
никчего не слышал про такие случаи, можно ли поподробнее

Doctor_D

это в нашей стране?
никчего не слышал про такие случаи, можно ли поподробнее
Завались таких случаев. Навскидку:
1. (Самый первый из мне известных.) Ярославль, начало 90-х. Девочка, лет 12-14, упала с качелей, ударилась плечом. Обратилась с матерью в больницу. Была осмотрена травматологом. Поставлен диагноз: ушиб мягких тканей. Обезболивание, фиксирующая повязка. Оставлена для дообследования. Дело было в пятницу вечером. Рентгенолог уже закончил работу. Снимок сделали в понедельник. На снимке - вколоченный перелом ключицы.
(Клиника - идентичная ушибу. Лечение (первичное) - аналогичное.)
Родители подали в суд за халатность (!) с требованием компенсации (примерно равной цене квартиры на тот момент). Выиграли дело в районном суде. В областном суде приговор отменили, но уже были задействованы личные связи одного хорошего человека из мед. академии.
Позднее этот человек рассказывал нам, что такие случаи - "развода" врачей случаются регулярно.
2.Тогда же. Там же. Новый Год. Группа студентов 5-го курса отмечала праздник. Пришла бабка-соседка, попросила помочь деду у которого стало плохо с сердцем. Студенты объяснили, что они изрядно пьяные и врядли смогут помочь. Посоветовали вызвать "скорую". Дед скончался от инфаркта. Двух студентов осудили за неоказание помощи - условный срок и лишение права заниматься медициной.
3.Магадан, 2000 г. Один из докторов, работавших в нашей фирме лечил на дому алкоголика. Т.наз "прерывание запоя". На 3-й день жена пациента пришла и потребовала деньги обратно + компенсацию, т.к муж опять запил и у него после капельницы развился флебит (воспаление вены). Мои объяснения, что а)прерывание запоя не гарантирует его возобновления, особенно, при игнорировании рекомендаций, б)флебит является прогнозируемым осложнением и не связан с квалификацией врача, в) а) и б) были им известны и прописаны в подписанном ими Договоре оказания услуг - были проигнорированы. Тетка обратилась в облздрав и лицензионный департамент. Было две проверки и несколько экспертиз. Ценой кучи времени и нервов ситуация была урегулирована.
4. Там же. 1998г. Врач "Скорой" приехал на вызов (инфаркт). Больной отказался ложиться на носилки и сам пешком спустился с 5-го этажа. Умер в больнице. Доктор получил 2 года общего режима за халатность.
5. Там же, 2001г. В хирургию доставили подростка с диагнозом аппендицит. Хирург еще до осмотра успокоил родителей что все ерунда и попросил привезти через пару часов бутылку коньяка - "отметить успех операции".
Мальчик умер еще до операционного стола - "Скорая" привезла его в критическом состоянии. У него оказалась редкая аномалия кишечника, приведшая к его разрыву и перитониту. Дней 5 лечился в поликлинике с диагнозом пищевое отравление. После второго осмотра врач поликлиники что-то заподозрила (ребенок был уже совсем плох) и направила его к хирургам.
Когда я уезжал - дело еще продолжалось, но папаша погибшего твердо пообещал посадить хирурга (хотя, 90% вины лежало на поликлинике и 10% на "Скорой"). Учитывая папашины деньги и связи - шансов оправдаться у хирурга не было. На момент моего отъезда его уже лишили категории и отстранили от работы.

десант

то есть в основном по смертельным случаям статистика.

Doctor_D

то есть в основном по смертельным случаям статистика.
Они лучше запоминаются. Да и менее серьезные случаи обычно без суда решаются. На уровне больницы или Управления здравоохранения.

Была помню, одна "профессиональная больная" - здоровая такая кобыла лет 40. Считала себя насквозь больной и постоянно ходила по врачам, изводя инстанции жалобами (санитарка не так посмотрела, медсестра больно укол сделала...). Проверки и комиссии потом просто с ума сводили.
Ну, так я посоветовал ей ректоскопию каждый день делать. На третий день выписалась добровольно... 😊
Это я к тому, что и пациенты и врачи разные бывают. 😊

десант

что то мы с вами в разные больницы ходим.

Doctor_D

что то мы с вами в разные больницы ходим.
Хм. Вы по моему, писали конкретные примеры удачного решения вопроса с плохими врачами. Чего же еще нужно? Чтобы все врачи стали идеальными? Это нереально. Врач - специалист, оказывающий услуги. Если не брать в расчет сложность - аналогичен парикмахеру или слесарю-водопроводчику. Вы же не будете утверждать, что у парикмахеров нет ответственности? Или что не всякому водопроводчику можно позволить чинить трубы в квартире? Все пользуются услугами специалистов, стараются выбрать лучших. С врачами - тоже самое должно быть.

десант

писал.
почему я заплатив деньги еще должен ходить разбираться?
медецина слишком спецефическая область, обывателю легко навешать лапши на уши.
пошла бы у меня жена одна, то все было бы как обычно. это я такой упертый.
и все эти ситуациии в столице были, а сталкивался в небольших городах и сельских -полная засада там с ответственностью.
и хорошо, если случаи не экстренные.

Doctor_D

почему я заплатив деньги еще должен ходить разбираться?
Се ля ви. 😞 Этой проблеме много веков. 😞
медецина слишком спецефическая область, обывателю легко навешать лапши на уши.
Хм. Много есть таких областей. Не может человек во всем разбираться. Мне, к примеру, можно навешать лапши в автосервисе или в ремонте бытовой техники. (Немного выручают аналитический склад ума и знание прикладной психологии. 😊)
пошла бы у меня жена одна, то все было бы как обычно. это я такой упертый.
Ну, я думаю, теперь жену научили правильному поведению? Чем больше людей будут отстаивать свои права (в пределах разумного, конечно), тем быстрее закончатся плохие специалисты (во всех областях).
в небольших городах и сельских -полная засада там с ответственностью.
и хорошо, если случаи не экстренные.
А это скорее, от менталитета зависит. Многие люди там вообще не знают, что у них есть права. В крайнем случае - попытаются морду обидчику набить, сядут за хулиганство и будут вопить из-за решетки: "нет у нас справедливости!!"

TIR

Doctor_D
Больной отказался ложиться на носилки и сам пешком спустился с 5-го этажа
Ога... Я лично видел как такие больные "отказываются". Чтобы врачи носилки предлагали это нонсенс - юзается только когда не срабатывает "больной вы идите, а мы вас поддержим за руку".
Doctor_D это не много ответственности - это в меру. За халатность нужно отвечать. А кто виноват и на сколько %% айда в суде доказывать.
2й случай это вообще финиш. Уроды редкосные, к тому же ещё и алкаши.
3й случай просто бзик клиента = ответственности никакой небыло и быть не могло.

А про хирурга... Хех. У меня помню "острый живот" был лет в 6, так мне тоже такой весёлый хирург попался и сразу грит "ща млин все потроха вырежем и зашибись!!!".
Слава богу хоть мать медик - спорить стала (они ещё глаза по 5ть копеек сделали типа как с ними такими грамотными кто-то смеет спорить). Симптомы отличались, и им это доооолго пришлось обьяснять. В итоге млин меня лечили от того, с чем у меня реально были проблемы. (потом привлекли других врачей к спору и они оказались неправы, операцию отменили).
А аппендикс у меня остался 😀 Хотя тааак хотелось "резать" и "давай следующего, времени нет!". Ещё бы уши отрезали млин...

Но это частный случай, когда медики встречают отпор тех, кто "в теме". Про обычных пациентов я молчу - им можно годами лапшу вешать, скальпелем резать и они будут думать что идёт лечение, не понимая "почему же не проходит?"

Это работа?! Да это блин колбасный цех! Конвеер блин!

десант

Doctor_D
А это скорее, от менталитета зависит. Многие люди там вообще не знают, что у них есть права. В крайнем случае - попытаются морду обидчику набить, сядут за хулиганство и будут вопить из-за решетки: "нет у нас справедливости!!"
вот и мне приходится за счет аналитического склада ума и логики обдумывать действия врачей.
а ежели мне полохо и не думается?

а по поводу местячковых проблем вы не правы.
имел несколько раз опыт попыток обжалования действий разных структур в своем городе -никаих шансов.
просто как в болоте.

Dr. San

десант
то есть в основном по смертельным случаям статистика.

Ну почему же...
Вот и от меня пара копеек:

1. Год этак 96-й.
Врач, заходя в палату спросил "Кто здесь "Иванов"?". Иванов обиделся, что его назвали не по имени-отчеству и подал на врача в суд. Компенсация морального ущерба - в эквиваленте современным 400 тыр.

2. 97 год.
Врач беседовал с лежачим больным в палате. Собирал анамнез, заполнял ИБ.
Пациент подает на врача в суд, за "разглашение врачебной тайны".
(посколюку в палате были и другие больные - то они могли слышать беседу, стало быть врачебная тайна была нарушена).
Компенсация ущерба - эквивален современным 500 тыр., врачу год условно.

десант

не верю.

Dr. San

TIR
2й случай это вообще финиш. Уроды редкосные, к тому же ещё и алкаши.

Вы не правы.
Для таких случаев существует СМП.

Студенты обязаны были оказывать помощь, если ситуация грозила немедленной смертью (медлить нельзя ни секунды) или нет никакой возможности вызвать СМП (нет связи, в лесу, в глуши...). В этих условиях любой гражданин, не медик, обязан оказать помощь в меру возможности.

Имеется и другая сторона вопроса - если бы студенты пришли и дали деду валидол, а он все равно умер бы, студентов судили бы за преступную халатность или убийство по неосторожности.
Так как на момент 5 курса мед подготовка у них только сестринская, и лечить и ставить диагнозы они не имеют права. К тому же они были пьяные.

В чем студенты виноваты - только в том, что не вызвали СМП сами, понадеялись, что бабка сама вызовет.

Dr. San

десант
не верю.

Ваше право 😊
Место действия - Иваново.

десант

и что обычная больница?
просто были у меня попытки взыскать моральный ущерб за более серьезные проступки ,так там присудили 2 000 рублей.
чтоб за "имя -отчество" присудили такую сумму -не верю. скорее всего было еще что-нибудь.

Dr. San

десант
и что обычная больница?
просто были у меня попытки взыскать моральный ущерб за более серьезные проступки ,так там присудили 2 000 рублей.
чтоб за "имя -отчество" присудили такую сумму -не верю. скорее всего было еще что-нибудь.

Попробуйте сами пересчитать, сколько на тот момент составляла сумма 125 миллионов в нынешних...

ЗЫ. Все проще, на самом деле.
Один врач-неудачник и скандалист, которого поперли из здравоохранения за склочный характер, организовал "Общество защиты прав пациентов".
Подключил юристов и занялся "разводом" больниц по таким вот случаям. Насколько я в курсе - выиграно было 6 или 7 дел.

Формально, и в том и другом случае придраться не к чему.
В 1 - по закону пациент имеет право на уважительное отношение. Если доказать, что в результате "неуважительного отношения" пациент получил моральный ущерб, приведший к вреду для здоровья (разволновался, подскочило давления, стало плохо с сердцем) - могли и посадить...

Во 2 - врач обязан соблюдать врачебную тайну, и не разглашать сведения, полученные от пациента. Формально - действия врача привели к тому, что сведения о пациенте стали доступны третьим лицам.
А не отвечать на вопросы врача пациент не мог, так как опасался за своою жизнь и здоровье...

десант

я не про формальную сторону.
здесь все понятно.
это же надо со всех свидетелей показания снять и прочее.
вот просто такую картину в жизни не представляю, вот честно.

михон

десант
классная фраза. она означает только одно -что система медецинского образования развалена, так как все медики дипломированные специалисты и с такими качествами при нормальном обучении никогда бы не получили диплома.

Естественно развалена. Я имел счастье учить наших современных студентов. Нормальных очень мало, к сожалению. Но дипломы получают все. Вообще, поработав в скоропомощьной больничке, многих своих "коллег" хочется не просто лишить диплома, а убить нахрен. Безграмотных врачей- полно. И ладно бы просто безграмотных, так они ещё и думать не умеют и не хотят.

десант

и какие ваши выводы после этого?

vasia2009

>что делать человеку у которого все знакомые и друзья не имеют отношения к медецине?

итти в монастырь, скит - тама научат...

михон

десант
и какие ваши выводы после этого?

Я их уже выше писал. Никогда не пойду к неизвестному мне лично или не рекомендованному знакомыми "коллеге". Потому как хоть за деньги, хоть бесплатно, высока вероятность "нарваться" на идиота. По той же причине никогда не пойду в ряд московских лечебных учреждений в принципе. Уж больно негативный опыт общения с "коллегами" есть. Хотя всё-таки приличные врачи есть. И их много. Если конкретная помощь нужна- пишите в личку.

десант

спасибо за предложение.
пока обходимся профилактикой болезней.
просто у меня ближайший рекомендованный и проверенный педиатр в 200 км от ребенка. (это я так для примера привожу)
по лекарствам проще -сестра фармацевт.
по медикам сложнее, но есть жены друзей-медики, есть друг с медобразованием.
но это все на уровне рекомендаций по телефону.
сам общаюсь с медиками в МСС, но опять же не тот уровень.
меня больше напрягает командировочная работа в этом плане.

vasia2009

>пара копеек:

ет просто метод выколачивания денег из государства - кому-то сподручнее ет делать через медсистему... тама ж не лично с семьи врача сняли то бабло...

>вот просто такую картину в жизни не представляю, вот честно.

ет ремесло - постигаеццо долгими годами...

Dr. San

vasia2009
>пара копеек:

ет просто метод выколачивания денег из государства - кому-то сподручнее ет делать через медсистему... тама ж не лично с семьи врача сняли то бабло...

Выплатила больница, естественно.
А к врачу - регрессный иск...

TIR

Dr. San
Студенты обязаны были оказывать помощь, если ситуация грозила немедленной смертью
Обазаны/нет... Это суд пусть установит.
Я делать то чего не знаю не буду. Но если меня позвут на помощь я подойду и помогу. Хотя бы своим присутствием - помощь в таких случаях часто бывает нужна.
Если человек вышел просить о помощи я думаю это не просто так.
Моральные кач-ва этих "медиков" мне чужды 😞

Doctor_D

Но если меня позвут на помощь я подойду и помогу. Хотя бы своим присутствием - помощь в таких случаях часто бывает нужна.
Вам хорошо. А врач в этом случае попадает под суд по заявлению "... эта пьяная сволочь своим неадекватным лечением довела нашего дорогого дедушку до смерти."
Один врач-неудачник и скандалист, которого поперли из здравоохранения за склочный характер, организовал "Общество защиты прав пациентов".
Слышал, кажется. Кстати, это - не единичный случай. Сейчас, правда, поутихли. Видимо, на автоподставы переключились.

Dr. San

TIR
Обазаны/нет... Это суд пусть установит.
Я делать то чего не знаю не буду. Но если меня позвут на помощь я подойду и помогу. Хотя бы своим присутствием - помощь в таких случаях часто бывает нужна.
Если человек вышел просить о помощи я думаю это не просто так.
Моральные кач-ва этих "медиков" мне чужды 😞

А в чем проблема-то?
Студенты РЕАЛЬНО оказали помощь - посоветовали бабке набрать 03 и вызвать СМП.

Еще раз повторю. Даже не вина студентов, а просто непредусмотрительность - не вызвали СМП САМИ, не надеясь на бабку. Но с учетом их упитости на момент обращения к ним - винить их можно только за вполне законом разрешенное пьянство. Не более того. В данном случае пьянство - "облегчающий признак". Так как они пили не на дежурстве, а в пьяном виде "лечить" не имели права...

Вы просто плохо представляете себе специфику медицинской подготовки.
Чем по-Вашему принципиально отличаются пьяные студенты-медики от пьяных студентов - энергетиков? Или пьяных студентов-энергетиков тоже можно посадить, если они по первому обращению откажутся исправить проводку в щитке на 50 кв?

Возможно, они и могли реально оказать помощь. Т.е. могли сказать, какую таблетку съесть, чтобы "стало лучше". И если, по счастливой случайности, деду бы повезло - он остался бы жив. Но если бы дед тем не менее помер - студентов законопатили бы по гораздо более серьезной статье. Ибо назначать лечение при инфаркте они не имели права по определению. Для определения этого диагноза и назначения лечения нужен трезвый врач с дипломом и ЭКГ.
И замечу, что "эффект присутствия пьяного студента-медика" мало помогает лечению инфаркта.

Более того, почему-то наказали только двоих студентов из всей пьянствовавшей группы. (как правило - группа человек 10-12).

Не наводит на размышления?


ЗЫ. Для примера - ситуация.
Едет в поезде врач на курорт. В соседнем купе пассажиру становится "плохо".
Врач говорит - а нах все пошли, я в отпуске.
(Первый вариант).

Тот же врач, тот же поезд. Пассажиру в соседнем купе плохо.
Но ньюанс - врач уже принял на грудь грамм 400. (А что, не имеет права?). И резонно предлагает остановить поезд на ближайшей станции и вызвать СМП. (Второй вариант).

Более того, пример про врача с дипломом, а не про студента 5 курса, который ОФИЦИАЛЬНО имеет право только укол сделать ПО НАЗНАЧЕНИЮ ВРАЧА, или клизьму поставить...

CyberDaemon

Думал долго писать или нет. Решил всетаки написать.

Как рассказывал мой инструктор по медицине (фамилия Бубнов) - помагать надо всегда. Даже если перед тобой уже труп с "селедочными" зрачками. Простоя показуха оборачивается душевным спокойствием, что сделал все что мог. И вот вопрос такой возник - Допустим сбили человека - ты видишь что он уе клинически мертв и рядом стоят его родственники, что будете делать? Пытаться показать что его откачиваете до приезда "скорой" или скажите в лицо родственникам что он "того"?

Dr. San

CyberDaemon
Думал долго писать или нет. Решил всетаки написать.

Как рассказывал мой инструктор по медицине (фамилия Бубнов) - помагать надо всегда. Даже если перед тобой уже труп с "селедочными" зрачками. Простоя показуха оборачивается душевным спокойствием, что сделал все что мог. И вот вопрос такой возник - Допустим сбили человека - ты видишь что он уе клинически мертв и рядом стоят его родственники, что будете делать? Пытаться показать что его откачиваете до приезда "скорой" или скажите в лицо родственникам что он "того"?

Вопрос в другом.
Вы еле держитесь на ногах по причине опьянения.
Пытаетесь оказывать помощь, хотя клиент уже мертв.
На Вас подают в суд, что Вы убили "любимого родственника" по причине своего безмерно пьяного состояния.

Вам легче будет?

ЗЫ. Кстати, я учился в институте имени Бубнова 😊
Есстественно, если собственное состояние позволяет - надо постараться сделать все возможное.
Но к случаю с инфарктым дедом и пьяными студентами-медиками это отношения не имеет.

Полагаю там было так:
Пришла бабка, сказала, что деду плохо. Ей посоветовали вызвать 03.
И продолжили квасить.
Бабка обиделась, и похерила вызывать 03. Деду поплохело, и он пример.
Бабка, чтобы оправдать свою вину, особенно перед своими детьми, обвинила студентов в неоказании помощи.
Студентов осудили, бабка в шоколаде, вся из себя праведница.
Ее детки не в обиде на бабку. Студенты же виноваты...
Всем хорошо...

CyberDaemon

Всем хорошо...

Кроме студентов.

Dr. San

CyberDaemon

Кроме студентов.

О чем и весь топик...

Doctor_D

TIR
Слава богу хоть мать медик - спорить стала (они ещё глаза по 5ть копеек сделали типа как с ними такими грамотными кто-то смеет спорить). Симптомы отличались, и им это доооолго пришлось обьяснять. В итоге млин меня лечили от того, с чем у меня реально были проблемы. (потом привлекли других врачей к спору и они оказались неправы, операцию отменили).
А аппендикс у меня остался 😀 Хотя тааак хотелось "резать" и "давай следующего, времени нет!". Ещё бы уши отрезали млин...
Мама хирург? Повезло, значит. Поверьте, больше всего дежурному хирургу хочется поспать, а не резать пятый за сутки аппендикс. Если настаивал - были основания. Просто у хирургов свой взгляд на "острые животы". Они справедливо считают, что в сомнительном случае лучше отрезать здоровый отросток, чем потом лечить перитонит (лапаротомия с разрезом от грудины до лобка, кишки промыть в тазике с антисептиком, засунуть обратно, через день повторить и надеяться, что спаечная болезнь будет не слишком тяжелой.)

михон

Doctor_D
в сомнительном случае лучше отрезать здоровый отросток, чем потом лечить перитонит
+500. Типичная аппендэктомия у меня занимала 20 минут. Зато одной головной болью за сутки меньше. Особенно если поступает человек 30 и всю ночь у стола.

TIR

михон
Типичная аппендэктомия у меня занимала 20 минут. Зато одной головной болью за сутки меньше.
То что вам так удобнее я даже и не сомневался 😀 Я ж и говорю - конвеер 😞

ЗЫ Почитал недавно книжку по военной медицие вообще в ужас пришёл. Вот уж где настоящий конвеер, но гражданская медецина далеко не ушла имхо. Во всяком случае та, что представлена "электорату".

Doctor_D

Почитал недавно книжку по военной медицие вообще в ужас пришёл.
Кстати, классическая задачка из военной медицины: вот вас, TIR, поставили на сортировку раненых. В операционной освободился стол. Перед вами три бойца (все тяжелые):
1)Раненый в голову.
2)Раненый в живот.
3)Раненый в ногу.
Кого из них вы отправите на операцию первым?
(Коллег прошу не отвечать - я знаю, что вы знаете. 😊)

TIR

Doctor_D
1)Раненый в голову.
2)Раненый в живот.
3)Раненый в ногу.
Кого из них вы отправите на операцию первым?
Без понятия. Я же не врач.
Могу предположить что работа будет организована так, чтобы пока оперируют одного не загнулись двое других.

михон

TIR
Могу предположить что работа будет организована так, чтобы пока оперируют одного не загнулись двое других.
Просто чтоб обычные люди понимали: глобально - задача военной медицины - вернуть в строй максимальное количество раненых. И этим она в корне отличается от гражданской. Для того, например, и существуют радиопротекторы и другие интересные препараты. Они не спасают жизнь бойцу, получившему огромную дозу радиации. Но дают возможность какое-то время даже смертельно поражённому солдату продолжать выполнять свой долг.
Для того и существует понятие медицинская сортировка. Когда помощь оказывается в первую очередь тому, у кого больше шансов вернуться в строй. Цинично? Может быть. Но в условиях войны от этого зависит жизнь всей страны. Однако в современных условиях, в локальных конфликтах, акцент всё-таки сместился в более гуманную сторону. И сортировка в классическом виде существует только при массовых поступлениях из очага санитарных потерь при нехватке квалифицированного медперсонала. В 95-ом такое было. Сейчас уже нет.

Торус

Doctor_D
Кстати, классическая задачка из военной медицины: вот вас, TIR, поставили на сортировку раненых. В операционной освободился стол. Перед вами три бойца (все тяжелые):
1)Раненый в голову.
2)Раненый в живот.
3)Раненый в ногу.
Кого из них вы отправите на операцию первым?
(Коллег прошу не отвечать - я знаю, что вы знаете. 😊)

Я не ТИР, но все равно попробую.
😊

Смотря по какому критерию отбирать.

Если в интересах гуманности, то:
1. Живот (перитонита в голове не бывает, и внутреннее кровотечение)
2. Голова (давящую повязку на шею 😛)
3. Нога

В интересах войны:
1. Нога
2. Живот
3. Голова


Dr. San

Торус
В интересах войны:
1. Нога
2. Живот
3. Голова

ИМХО, неправильно.
1. В любом случае - сначала "нога". Высокая вероятность выживания.
2 и 3. Вопрос дискутабельный. Зависит от квалификации медперсонала.

В любом случае, при низкоквалифицированном персонале по-любому - живот. Потому что ранение в живот хоть и приводит в 95% случаев к перитониту, нейрохирургическая операция - почти 100%-я смерть.

Но если персонал квалифицированный - тогда по-любому, сначала голова.
А уже потом живот.

михон

На самом деле всё зависит от характера ранения. В отношении живота: если это минно-взрывное, с эвентрацией и прочими прелестями, тяжёлый шок и т.д. - в условиях большой войны - однозначно в палатку агонирующих. А какую-нибудь лёгкую голову - на трепанацию в первую очередь. Если нога с повреждением сосудисто-нервного пучка на бедре и профузным артериальным кровотечением - в первую очередь, хотя бы временный силиконовый шунт воткнуть и нитками закрепить. В общем, всё зависит не только от анатомической зоны, но от характера ранения, общего состяния раненого, этапа эвакуации, подготовки и количества персонала, возможностей дальнейшей эвакуации по предназначению и т.д.
А вообще, может в Медицину поедем, а? Или в Армейский?

Dr. San

михон
На самом деле всё зависит от характера ранения. В отношении живота: если это минно-взрывное, с эвентрацией и прочими прелестями, тяжёлый шок и т.д. - в условиях большой войны - однозначно в палатку агонирующих. А какую-нибудь лёгкую голову - на трепанацию в первую очередь. Если нога с повреждением сосудисто-нервного пучка на бедре и профузным артериальным кровотечением - в первую очередь, хотя бы временный силиконовый шунт воткнуть и нитками закрепить. В общем, всё зависит не только от анатомической зоны, но от характера ранения, общего состяния раненого, этапа эвакуации, подготовки и количества персонала, возможностей дальнейшей эвакуации по предназначению и т.д.
А вообще, может в Медицину поедем, а? Или в Армейский?

Ну, с хирургами я спорить не буду. Согласен полностью.
Вот на тему мозгов - другое дело 😊

James Bond

михон
А вообще, может в Медицину поедем, а? Или в Армейский?
Как скажешь!


перемещено в Медицина